JULY 2018 Pennsylvania Bar Examination: Essay Questions and Examiners' Analyses and Performance Test

You might also like

Download as pdf or txt
Download as pdf or txt
You are on page 1of 82

JULY 2018

PENNSYLVANIA BAR
EXAMINATION

Essay Questions and Examiners’ Analyses


and
Performance Test

Pennsylvania Board of Law Examiners


601 Commonwealth Avenue, Suite 3600
P.O. Box 62535
Harrisburg, PA 17106-2535
(717) 231-3350
www.pabarexam.org

©2018 Pennsylvania Board of Law Examiners


Table of Contents

Index ..................................................................................................................................................ii

Question No. 1: Facts and Interrogatories, Examiner's Analysis and Grading Guidelines ..............1

Question No. 2: Facts and Interrogatories, Examiner's Analysis and Grading Guidelines ..............8

Question No. 3: Facts and Interrogatories, Examiner's Analysis and Grading Guidelines ..............16

Question No. 4: Facts and Interrogatories, Examiner's Analysis and Grading Guidelines ..............24

Question No. 5: Facts and Interrogatories, Examiner's Analysis and Grading Guidelines ..............35

Question No. 6: Facts and Interrogatories, Examiner's Analysis and Grading Guidelines ..............46

Performance Test and Grading Guidelines ........................................................................................55

i
Index

Question No. 1

1. Decedent’s Estates: undue influence

2. Decedent’s Estates: precatory words

3. Federal Income Tax: gambling income

4. Professional Responsibility: communication with represented person

Question No. 2

1. Business Organization: promoter liability, piercing corporate veil

2. U.C.C. Article II: limitation on remedies repair and replace

3. U.C.C. Article II: limitation on remedies consequential damages

Question No. 3

1. Criminal Law: involuntary manslaughter

2. Evidence: prior bad acts

3. Criminal Procedure: inventory search

4. Family Law: divorce

Question No. 4

1 Employment Law: disparate impact

2. Civil Procedure: class certification

3. Constitutional Law: mootness

ii
Question No. 5

1. Contracts: formation, counter-offer

2. Property: tenancy in common, joint tenancy with right of survivorship

3. Contracts: mitigation of damages

4. Property: equitable servitude

Question No. 6

1. Torts: professional negligence, vicarious liability

2. Civil Procedure: statute of limitations

3. Evidence: subsequent remedial measures

4. Torts: comparative negligence

iii
Question No. 1

Mike, a resident of B County, Pennsylvania, is 65 years old and single. Mike and his son,

Joe, were avid fans of the Pennsylvania Dogs professional football team. During football season,

they attended Dogs tailgate parties and games on Sundays, often drinking alcohol throughout the

day. Neither Mike nor Joe was observant of any religious denomination, and Mike was fond of

saying “football is our religion.” In addition to watching games, Mike routinely placed large

bets on the Dogs and other football teams, and he had net winnings of $40,000 from gambling on

football games during January 2018. On February 1, 2018, Mike placed a bet of $10,000 that the

Dogs would win their final game of the season. The Dogs lost 35-0, and Mike lost his $10,000

wager.

On February 5, Mike was diagnosed with stomach cancer and advised that he had only a

few months to live. In March, Mike met Robin, the pastor at the New Pennsylvania Church

(“Church”). Robin invited him to attend services that Sunday, which he did. From that day on,

Mike attended services at Church every week, and met with Robin twice a week for counseling.

Joe observed that his father remained cheerful and organized in his daily life, and Mike often

expressed his gratitude to Robin for helping him find peace.

Mike met with Lisa, an attorney and member of Church who also happened to be a friend

of Joe’s. Lisa prepared a new will, which specifically revoked Mike’s old will, which had

provided for his entire estate to pass to Joe. The new will read, in pertinent part, as follows:

I give the entirety of my estate as follows: one-half to Robin, and one-half to my


son Joe, to be used as Joe chooses. It is my wish that Joe stop drinking and attending
football games on Sundays and begin attending church and living a holy life. If
Robin or Joe predeceases me, his or her share shall pass to New Pennsylvania
Church. I name Robin as executor.

1
Mike properly executed the will on April 20, 2018, and properly signed a general durable power

of attorney naming Robin as his agent. On May 1, 2018, Mike died. Robin probated Mike’s will

on May 10, 2018.

Joe retained Andy, an attorney, to file a challenge to the will on the basis of undue

influence. Andy timely filed Joe’s claim, and a hearing date of August 15, 2018, was set. Robin

engaged Lisa as her counsel. Shortly thereafter, Joe approached Lisa at a social gathering and

brought up the case, but Lisa said “Joe I can’t talk about this because you have a lawyer.” They

spoke for approximately twenty minutes about unrelated topics. Afterwards, Lisa called Andy to

inform him that she had spoken to Joe but did not discuss the case; Andy replied, “Feel free to

talk with Joe about the case, I think everyone would be well served if we can settle.” In July,

after incurring substantial legal fees in connection with hearing preparations, Joe saw Lisa at a

restaurant and confronted her, saying “We used to be friends! This case is bankrupting me and

tearing my family apart!” Lisa replied, “Joe, I was there when your dad signed the will, and I

believe it was what he wanted. Isn’t there some way we can work this out?” Joe walked away.

Meanwhile, a new football season began and Joe resumed his practice of attending

games, drinking, and partying on Sundays. Church filed a petition asking the court to rule that

Joe forfeited his share of the estate when he violated the wishes contained in Mike’s will.

1. How is the court likely to rule on Joe’s undue influence claim?

2. Assume for this question only that the will was held to be valid. How will the
court rule on Church’s petition to divest Joe of his share in Mike’s estate?

3. What, if any, are the federal income tax consequences of Mike’s gambling during
calendar year 2018? Assume that Mike is a cash basis taxpayer, calendar year tax
filer, entitled to itemize deductions and that all of his 2018 gambling transactions
were legal and are accounted for in the fact pattern.

4. Did Lisa violate the Pennsylvania Rule of Professional Conduct governing


communications with represented persons in her interactions with Joe?

2
Question No. 1: Examiner’s Analysis

1. The court is likely to deny Joe’s claim that Robin exerted undue influence over Mike
in the drafting of his will because Joe cannot establish that Mike had a weakened
intellect.

“The resolution of a question as to the existence of undue influence is inextricably linked


to the assignment of the burden of proof.” Estate of Clark, 461 Pa. 52, 59, 334 A.2d 628, 631
(1975). After the will is admitted to probate, the burden is on the challenger to the will to
establish undue influence. Id. To establish a prima facie case for undue influence, a contestant to
a will must show the following factors: (1) confidential relationship; (2) substantial benefit; and
(3) weakened intellect. Estate of Reichel, 484 Pa. 610, 615, 400 A.2d 1268, 1270 (1979). If
these elements are proven by clear and convincing evidence, the burden of proof shifts to the
proponent of the will to show that the bequest was not the product of undue influence. Id.

A confidential relationship exists when the “parties did not deal on equal terms, but on
the one side there is an overmastering influence, or, on the other, weakness, dependence or trust,
justifiably reposed.” Estate of Clark, 461 Pa. at 63, 334 A.2d at 633 (quotation and citation
omitted).

[A] confidential relationship is not limited to any particular association of parties


but exists wherever one occupies toward another such a position of advisor or
counsellor as reasonably to inspire confidence that he will act in good faith for the
other's interest.

Brooks v. Conston, 356 Pa. 69, 76, 51 A.2d 684, 688 (1947). “[N]o clearer indication of a
confidential relationship can exist than giving another person the power of attorney over one’s
entire life savings.” Estate of Bankovich, 344 Pa. Super. 520, 524, 496 A.2d 1227, 1229 (1985)
(quotation and citation omitted).

It is likely that Joe can establish that Robin stood in a confidential relationship with Mike.
Her relationship with Mike was rooted in her capacity as his pastor, and she provided counseling
to Mike at a difficult time in his life, i.e. when he was facing sickness and death. She was also
named his agent under a power of attorney. Mike exhibited trust in and showed dependence on
Robin, and Robin was in a position of authority with respect to Mike.

It is also likely that Joe can establish that Robin received a substantial benefit from the
will. Whether the proponent of the will received a substantial benefit is determined on a case by
case basis. Estate of Fritts, 906 A.2d 601, 609 (Pa. Super. 2006). In Fritts, the court stated that
an increase from a 40% share to a 61.5 % share constituted a substantial benefit. Id. Merely
being named executor is not enough to establish a substantial benefit. Estate of Stout, 746 A.2d
645, 649 (Pa. Super. 2000). However, it is relevant whether the proponent of the will is a
relative by blood of the decedent Id. (“[W]here there is a blood relationship between the testator
and the beneficiaries of her estate, that fact alone forms a sufficient, independent basis for the
bequest.” (citation and internal quotation omitted)). Here, Robin is not a relative of Mike and

3
has been named executor. Moreover, she had no interest under his prior will, and received a one-
half interest under the will at issue. All of these factors together likely mean Robin received a
substantial benefit from Mike’s 2018 will.

Joe is unlikely to be able to establish the final element of undue influence, however.
Pennsylvania law has “not established a bright-line test by which weakened intellect can be
identified to a legal certainty,” however, “it is typically accompanied by persistent confusion,
forgetfulness and disorientation.” Estate of Fritts, 906 A.2d 601 at 607. “The weakened mental
condition which must be shown does not [need to] rise to the level of testamentary incapacity.”
Estate of Ziel, 467 Pa. 531, 542, 359 A.2d 728, 734 (1976). “Evidence of physical infirmities,
however, is not enough, alone, to establish weakened intellect.” Estate of Glover, 447 Pa. Super.
509, 517, 669 A.2d 1011, 1015 (1996).

Bodily weakness alone is not sufficient to shift the burden of proof upon the
beneficiary occupying a confidential relation to the testatrix, for one may be
physically weak and yet have a perfectly sound and strong mind.

King’s Will, 369 Pa. 523, 529-30, 87 A.2d 469, 473 (1952).

There is nothing in the facts to suggest that Mike’s mental state approached that required
to show weakened intellect. His bodily illness may have caused pain, discomfort, and stress but
there is no evidence that it affected his mental state. Nothing in the facts suggests that Mike was
confused, disoriented, or forgetful; in fact, Joe observed that Mike remained cheerful and
organized in his daily life. Because Joe cannot establish a weakened intellect, he will fail to
establish a prima facie case for undue influence. As a result, the court will likely deny Joe’s
claim.

2. The court will likely deny the petition to divest Joe of his share of Mike’s estate
because Mike’s wishes regarding Joe’s conduct as expressed in his will are
precatory.

The language in Mike’s will indicating his desire that Joe begin attending church and stop
drinking and watching football is likely to be interpreted to be precatory and not binding.
“Precatory words have been defined as words in a will expressive of desire, recommendation and
confidence.” In re Kirk’s Estate, 38 Pa. D. & C.2d 532, 534 (Pa. Com. Pl. 1965) (quotation and
citation omitted); see also The Wolters Kluwer Bouvier Law Dictionary Desk Edition (2012)
(“Language in an instrument that requests some conduct rather than commanding it.”). Mike’s
stated “wish” is not phrased as mandatory.

[T]he word "wish" . . . is generally classified as precatory. However, such a word


may be mandatory when expressive of an intention of the testator to be carried out
without the intervention of another's will and when used "in direct reference to the
estate."

In re Corbett’s Estate, 430 Pa. 54, 57, 241 A.2d 524, 525 (1968) (quoting Calder’s Estate, 343
Pa. 30, 37, 21 A.2d 907, 911 (1941).

4
While expressions of a wish will amount to a testamentary disposition if such is in
accordance with the intention of the testator, yet, where precatory words follow an
absolute disposition of the property, it has been established as a rule of property in
Pennsylvania that such precatory words will not operate to diminish the absolute
estate previously given.

Chew v. Chew, 266 Pa. 526, 528, 109 A. 799, 800 (1920); see also Sommerville Estate, 417 Pa.
600, 605, 209 A.2d 299, 302 (1965) (“An absolute estate will not be cut down by subsequent
language unless the testator has expressed a clear and unambiguous intention to do so.”).

Mike’s gift to Joe is absolute – the will specifically states that Joe may do with the
bequest “as he chooses.” The will also provides that if Joe predeceases Mike, his share goes to
New Pennsylvania Church, but does not require the same result if Joe fails to follow his
expressed wishes. Moreover, Mike’s expression of desire for his son’s future conduct does not
directly mention his estate. Therefore, the general interpretation of “wishes” will control, and
the court will likely determine that the words regarding Joe’s behavior are precatory.
Accordingly, the court will dismiss Church’s petition.

3. Mike has taxable income of $30,000 from gambling in 2018.

Gross income generally includes “all income from whatever source derived.” 26 U.S.C.
§ 61. This includes income from gambling winnings. See e.g. Carmack v. Commissioner, 183
F.2d 1, 2 (5th Cir. 1950) (stating poker winnings to be included in taxable income); Bodine v.
Commissioner, T.C. Memo 1984-143 (holding gambling winnings included in income even
where tax was withheld). “Losses from wagering transactions shall be allowed only to the extent
of the gains from such transactions.” 26 U.S.C.S. § 165(d); 26 C.F.R. § 1.165-10.

Mike had a net $40,000 in gambling winnings prior to the football championship game,
and he lost $10,000 on that game. These transactions represent all of Mike’s gambling activity
for the year. Under section 61 Mike will have to report his net $40,000 in gambling winnings on
his 2018 taxes.

Then, pursuant to section 165(d), Mike will be entitled to deduct wagering losses from
his gambling gains. The question indicates that Mike may itemize his deductions for the
calendar year. After deducting his championship game loss of $10,000 from his gambling gains
of $40,000, Mike is left with $30,000 of taxable income from gambling transactions for calendar
year 2018. The federal tax consequences of Mike’s gambling activity are income of $40,000,
and a deduction of $10,000, or net income of $30,000.

4. Lisa did not violate the Pennsylvania Rule of Professional Conduct in her
communications with Joe.

The rules of Professional Conduct regulate communication between lawyers and


represented parties. “In representing a client, a lawyer shall not communicate about the subject

5
of the representation with a person the lawyer knows to be represented by another lawyer in the
matter, unless the lawyer has the consent of the other lawyer or is authorized to do so by law or a
court order.” Pa. R.P.C. 4.2. “This Rule does not prohibit communication with a represented
person, or an employee or agent of such a person, concerning matters outside the representation.”
Pa. R.P.C. 4.2 cmt. 4. “The Rule applies even though the represented person initiates or consents
to the communication.” Pa. R.P.C. 4.2 cmt. 3.

The first time Lisa spoke with Joe after becoming involved in the case, she was aware
that Joe was represented by Andy, as she demonstrated by telling Joe she could not discuss the
case. As such, she did not communicate with him about the subject of the representation. The
first communication was permissible under Rule 4.2.

By the time of the second communication, during which Lisa did communicate with Joe
about the subject of the representation, Andy had authorized Lisa to communicate with Joe about
the case. Therefore, Lisa did not violate any rule of professional conduct in either of her
communications with Joe.

6
Question No. 1: Grading Guidelines

1. Undue influence.

Comments: Applicants should identify and apply the elements required to state a prima facie
case for undue influence.

7 points.

2. Precatory words.

Comments: Applicants should recognize the rule that precatory words are not binding on the
beneficiaries of a will.

5 points.

3. Taxation of gambling gains and losses.

Comments: Candidates should recognize that gambling gains are included in gross income, and
that gambling losses can be deducted.

3 points.

4. Communication with a represented person.

Comments: Candidates should identify the rule prohibiting communication with a represented
person concerning the subject matter of the representation and the exception where counsel for
such person gives consent to the communication.

5 points.

7
Question No. 2

In January 2018, Diane decided to open a commercial photography studio from which

she could take photographs for sale and could produce custom posters for events. Subsequently,

Diane decided to form a corporation, Photos, Inc. (“Photos”), to be the operator of the business.

In January, Diane negotiated a lease for a commercial space in Pennsylvania with Realty,

a Pennsylvania company. Diane told Realty she was acting on behalf of Photos. The fully

executed written lease named Photos as tenant and was signed by Diane as president of Photos.

In April, Diane, without consulting an attorney, incorporated Photos as a Pennsylvania

corporation. She is Photos’ sole shareholder. Photos has no bylaws, minutes, issued stock

certificates, letterhead or business cards. Diane did not contribute any assets to Photos other than

$200 in cash. She has been using a camera that she owns personally. Photos has a checking

account from which Diane routinely pays both Photos’ expenses and her personal expenses.

On July 1, Photos was awarded a contract to produce customized posters for each senior

athlete at a local Pennsylvania high school to be presented to the athletes at an assembly on the

first day of school in late August. The contract required delivery of the posters by August 20 and

expressly provided, that if all posters are not delivered by August 20, the contract would be void

and the school would have no obligation to pay Photos for any work done.

Diane concluded that Photos would need a new laser printer to produce the posters on a

timely basis. On July 2, she contacted Printers, Inc. (“Printers”), a manufacturer and seller of

printers and discussed Photos’ need for a printer for its commercial operation. Diane, as agent of

Photos, purchased a new laser printer. The written sales agreement, properly executed and

signed by Printers and Photos, provided, in part:

The buyer’s exclusive remedy against the seller for failure of or defect in the
printer shall be for the repair or replacement of defective parts. No other remedy

8
(including, without limitation, incidental or consequential damages for lost
profits, lost sales, or any other incidental or consequential loss) shall be available
to buyer. The buyer has signed this provision to evidence its understanding and
agreement that the remedy provided for is the sole and exclusive remedy available
to the buyer in the event of failure of the printer. Diane Buyer’s signature

The printer was delivered July 10, set up, tested and accepted by Photos. The printer was

initially working fine but, three days ago, stopped working. Diane promptly called Printers, and

Printers immediately sent out a technician. It was determined that the circuit board in the printer

was defective and needed to be replaced. Diane demanded a new printer or her money back

because the printer provided had failed in breach of the warranty of merchantability. Printers

refused and advised it would promptly install a new circuit board in the printer. Diane,

concerned about the delay the repair of the printer would cause, contacted the school and advised

the posters would not be delivered until August 30. The school indicated that if all of the posters

were not delivered by August 20 the contract would be void. Printers installed a new circuit

board yesterday and the printer is again working fine.

Photos is really counting on the school contract as it has fallen behind in paying rent,

utilities, and other bills. Due to these events, Diane is contemplating going out of business.

1. If Photos ceases operations, what two theories could be advanced by Realty to


support a claim against Diane personally in an action to collect past due rent
based, first, upon Diane’s pre-incorporation activities and, second, her status as
shareholder of Photos?

2. Under the Pennsylvania Uniform Commercial Code (the “Code”), when Diane
demanded a replacement printer or her money back was Printers within its rights
under the contract to refuse her request and proceed with steps to repair and
replace the defective circuit board?

3. Assume for this question that Photos has not produced and delivered all of the
posters and the school validly declares the contract void per the contract terms.
Under the Code, can Photos successfully recover its lost profits from Printers if it
asserts and proves that its failure to meet the deadline was due to the failure of the
laser printer?

9
Question No. 2: Examiner’s Analysis

1. Realty could argue that Diane has liability under the lease as a promoter and could
also argue the equitable theory of piercing the corporate veil of Photos in an effort
to find Diane personally liable under the lease.

Realty could argue that when the lease was signed Diane was acting as a promoter of
Photos. A “promoter” is a person who purports to act on behalf of a proposed corporation that
has not yet been incorporated. Generally, a promoter is someone who takes an active part in
creating and organizing the corporation. An officer who executes or enters into a pre-
incorporation contract may have the legal status of a promoter. Fletcher Cyc. Corp. § 189 (Perm.
Ed. 1999).

In the absence of an express or implied agreement to the contrary, a promoter is liable on


a pre-incorporation contract even though the contract purports to be on behalf of the corporation
to be formed. Id. at § 215. “Even though they purport to act on behalf of the proposed
corporation and not for themselves, promoters may be held personally liable on contracts made
by them prior to the actual formation of the corporation. In the absence of a novation or an
agreement by the other party to a release of liability, the promoter will remain liable after the
corporation is formed.” John W. McLamb & Wendy C. Shiba, Pennsylvania Corporate Law
&Practice § 2.2 (1993).

With respect to the lease agreement with Realty, Diane was clearly acting as a promoter.
She negotiated the lease with Realty purporting to enter into the lease contract on behalf of a
corporation (Photos) that had not yet been formed. There is no indication that Realty agreed to
look only to Photos to enforce its rights under the lease or to release Diane if and when Photos
was formed. Realty should be successful in asserting a claim directly against Diane if it so
desires based upon her status as a promoter. See, RKO-Stanley Warner Theaters, Inc. v.
Graziano, 467 Pa. 220, 355 A.2d 830 (1976).

Realty could also argue that it should be permitted to pierce the corporate veil of Photos
and collect the past due rent directly from Diane as Photos’ sole shareholder.

Generally, a corporation is an entity separate and apart from its shareholders even if all of the
stock of the corporation is owned by one individual or entity. Barium Steel Corp. v. Wiley, 379 Pa.
38, 47, 108 A.2d 336, 341 (1954). Courts will uphold the separate corporate existence in most cases
and leave the shareholder free of liability for the corporation’s debts. John W. McLamb, Jr. & Wendy
C. Shiba, Pennsylvania Corporate Law & Practice, § 5.9 (1993). There are, however, certain
instances when the corporate veil will be pierced and the general rule of shareholder protection will
not apply.

There are no precise rules or tests to be applied to determine if the corporate veil should be
pierced. Barium Steel Corp., 379 Pa. at 47, 108 A.2d at 341. Piercing a corporate veil is equitable in
nature and is driven by the circumstances of each case. “There are, however, two [overarching]
elements required by most jurisdictions. First, there must be such unity of interest and ownership
that the separate personalities of the corporation and the individual no longer exist[. S]econd, the

10
circumstances must indicate that adherence to the fiction of separate corporate existence would
sanction a fraud or promote injustice.” Fletcher Cyc. Corp., supra § 41.30.

Although the analysis is somewhat subjective, there are general elements that have been
identified by courts that support a finding that the corporate veil should be pierced. These factors
include, whether the corporation is inadequately capitalized, failed to observe corporate formalities,
failed to issue stock, failed to pay dividends, operates without profit, commingles corporate assets,
with personal assets of its shareholder(s) and is devoid of corporate records. Id., see also, Lumax
Industries v. Aultman, 543 Pa. 38, 669 A.2d 893 (1995). The Pennsylvania Supreme Court stated,
“the corporate veil is properly pierced whenever one in control of a corporation uses that control or
corporate assets to further one’s own personal interests.” College Watercolor Group, Inc. v.
Newbauer, 468 Pa. 103, 117, 360 A.2d 200, 207 (1976). Thus, if the corporation exists merely in
name only and is used as the alter ego of the shareholder a court may pierce the corporate veil and
impose liability directly on the shareholder.

In this case, Photos has minimal assets. Diane has failed to observe any corporate
formalities. Photos has no bylaws, minutes, or issued stock certificates. It has no letterhead or
business cards. Even though it has a corporate checking account, Diane routinely uses funds in that
account to pay her personal expenses which acts to the detriment of Photos’ creditors. Based on
these factors, Realty has a basis to seek to have the corporate veil pierced, should the promoter
liability action prove unsuccessful, and may be successful in doing so.

2. Printers most likely had the contractual right to refuse to replace the failed printer
with a new one or to refund Photos’ money and to proceed with repair and
replacement of the defective circuit board.

When the printer failed, Diane demanded a new printer or her money back based upon a
belief that the failure of the printer constituted a breach of the implied warranty of
merchantability. She may be correct in her assertion of breach of warranty. The facts are silent
as to existence of any disclaimer of this implied warranty. The issue presented here, however, is
not one of the applicability of an implied warranty, which is most likely implicated by the facts;
but, instead is one of what remedies are available to the buyer as a result of a breach. If this
warranty was breached, normally, provided any and all procedural steps are properly taken,
rejection and return of the good and replacement or refund of the purchase price are remedies
that could be available under the Code. See 13 Pa. C.S.A. §§ 2601, 2608. Here, Printers’
position is that the contract defines the exclusive remedy available as the repair and replacement
of the defective part.

Section 2719 of the Code addresses this situation. This section provides:

(a) General rule .-- Subject to the provisions of subsections (b) and (c) and of
section 2718 (relating to liquidation or limitation of damages; deposits):

(1) The agreement may provide for remedies in addition to or in substitution for
those provided in this division and may limit or alter the measure of damages
recoverable under this division, as by limiting the remedies of the buyer to return

11
of the goods and repayment of the price or to repair and replacement of
nonconforming goods or parts.
(2) Resort to a remedy as provided is optional unless the remedy is expressly
agreed to be exclusive, in which case it is the sole remedy.

(b) Exclusive remedy failing in purpose. -- Where circumstances cause an


exclusive or limited remedy to fail of its essential purpose, remedy may be had as
provided in this title.

(c) Limitation of consequential damages. -- Consequential damages may be


limited or excluded unless the limitation or exclusion is unconscionable.
Limitation of consequential damages for injury to the person in the case of
consumer goods is prima facie unconscionable but limitation of damages where
the loss is commercial is not.

13 Pa. C.S.A. §2719.

Generally, limitations on remedies provided in a sales contract are optional unless the
agreement expressly provides that the remedy or remedies are to be exclusive. 13 Pa. C.S.A.
§2719(a)(2). Comment 2 to section 2719 provides,

Subsection (1)(b) creates a presumption that clauses prescribing remedies are


cumulative rather than exclusive. If the parties intend the term to describe the sole
remedy under the contract, this must be clearly expressed.

In this case, the contract clearly provides “the buyer’s exclusive remedy . . . shall be . . . repair
and replacement of defective parts.” This fact was made clear to the buyer as the contract
required the buyer to sign the limitation of remedy clause of the contract. On its face, it would
appear that this is the only remedy that the buyer can request and demand. The analysis,
however, does not end there. The remedy must be further analyzed under Subsection (b); i.e.,
does the remedy “fail of its essential purpose” and it must be determined if the remedy and the
exclusion of all other remedies is unconscionable.

Subsection (b) of 2719 provides, “Where circumstances cause an exclusive . . . remedy to


fail of its essential purpose, remedy may be had as provided in this title.” In other words, if the
remedy fails of its essential purpose then a court may determine that it is not the exclusive
remedy available to the buyer and that the buyer may resort to other remedies available under the
Code. Comment 1 to section 2719 provides, in part:

[I]t is the very essence of a sales contract that at least minimum adequate
remedies be available. If the parties intend to conclude a contract for sale within
this Article they must accept the legal consequence that there be at least a fair
quantum of remedy for breach of the obligations or duties outlined in the contract.

A remedy fails of its essential purpose if it fails to provide a remedy that gives the buyer the
benefit of its bargain and if it fails to do exactly what it is intended to do; i.e., repair the printer.

12
13 Pa. C.S.A. §2719, cmt. 1. Normally, the buyer must give the seller a reasonable opportunity
to effectuate the remedy and the seller must, in fact, effectuate the remedy curing the problem.
See White and Summers, Uniform Commercial Code, §12-10 a. (4th Ed. 1995). In other words
the buyer must give the seller notice of the problem and allow the seller an opportunity to
remedy the situation.

In this case, when the printer stopped working, the buyer notified the seller who
immediately sent out a technician who identified the defective circuit board as the problem.
Within days, the circuit board was replaced and the printer was again working as it should.
Although the determination of whether or not a remedy “fails its essential purpose” is looked at
on a case-by-case basis a strong argument can be made by Printers that the remedy was adequate
and did not fail in its essential purpose. The remedy did what it was supposed to do. It resulted
in the prompt repair of the printer. See New York State Elec. & Gas Corp. v. Westinghouse Elec.
Corp., 387 Pa. Super. 537, 557, 564 A.2d 919, 929 (1989). Should a court find otherwise, then
the exclusive nature of the remedy would fail and the buyer could resort to other remedies. See,
White and Summers, Uniform Commercial Code, § 12-10 (4th Ed. 1995).

Comment 1 to Section 2719 further provides “any clause purporting to modify or limit
the remedial provisions of this Article in an unconscionable manner is subject to deletion and in
that event the remedies made available by this Article are applicable as if the stricken clause had
never existed.” Section 2302, which addresses unconscionability, provides:

(a) Finding and authority of court.--If the court as a matter of law finds the
contract or any clause of the contract to have been unconscionable at the time
it was made, the court may:

(1) refuse to enforce the contract;

(2) enforce the remainder of the contract without the unconscionable clause; or

(3) so limit the application of any unconscionable clause as to avoid any


unconscionable result.

(b) Evidence by parties.--When it is claimed or appears to the court that the


contract or any clause thereof may be unconscionable the parties shall be afforded
a reasonable opportunity to present evidence as to its commercial setting, purpose
and effect to aid the court in making the determination.

Comment 1 to Section 2302 explains, “The basic test is whether, in the light of the general
commercial background and the commercial needs of the particular trade or case, the clauses
involved are so one-sided as to be unconscionable under the circumstances existing at the time of
the making of the contract.” Unconscionability is a somewhat subjective standard without clear
lines that delineate when something is or not unconscionable. A court would hear testimony and
would have to determine if the limitation of the remedy, as set forth in the contract, is or is not
unconscionable. In a commercial setting a court will rarely find a contract clause
unconscionable. “The principal underlying a finding of unconscionability is to prevent
oppression and unfair surprise . . . .” Jim Dan, Inc. v. O. M. Scott & Sons Company, 785 F.

13
Supp. 1196, 1200 (W.D. Pa. 1992). In this case, the remedy was clearly called out in the
contract and was acknowledged and accepted by the buyer. When a problem occurred, the
remedy proved to be effective, as the printer was repaired by replacement of the defective part
and was back up and running within a week. Although reasonable minds could disagree, a good
argument could be made that the limitation of remedy is not unconscionable and should be
upheld by a court.

3. A sales contract may limit the availability of consequential damages unless the
limitation or exclusion is unconscionable and, therefore, it is unlikely that Photos
will recover for its lost profits.

As set forth above, 2719(c) provides, “Consequential damages may be limited or


excluded unless the limitation or exclusion is unconscionable. Limitation of consequential
damages for injury to the person in the case of consumer goods is prima facie unconscionable but
limitation of damages where the loss is commercial is not.” 13 Pa.C.S.A. § 2719(c). In the
instant case we are not dealing with a sale of consumer goods to a consumer and the claim would
not be one asserted for injury to a person. Thus, the second sentence of 2719(c) would not be
applicable. This was a sale to a commercial buyer for use in a commercial enterprise.

The question asked is whether Photos could recover lost profits. Section 2715 of the
Code defines “consequential damages” to include “any loss resulting from general or particular
requirements and needs of which the seller at the time of contracting had reason to know and
which could not reasonably be prevented by cover or otherwise.” 13 Pa.C.S.A. § 2715(b).
Printers knew that the printer was being purchased by a commercial enterprise to print posters.
Thus, loss of profits due to the failure or inability to print posters was or should have been
known to Printers.

The question, then, is the effect of the exclusion of consequential damages in the
contract. Generally, commercial parties are and should be free to contract as they wish. See,
White and Summers, Uniform Commercial Code § 12-10 c. (4th Ed. 1995). This general rule, of
course, is tempered by the general application of the unconscionability provisions of the Code.
See Section 2302 discussion above. In this case, the parties clearly agreed to the limitation. This
case does not deal with a consumer transaction. Given the precept that parties, particularly
commercial parties, should be free to contract as they wish, a strong argument can be made that
the exclusion of consequential damages should apply and that Photos would not be able to
recover lost profits. A further argument could be made by Printers that the minimal down time
should not have prevented Photos from completing its project on time. Thus the lost profits
being claimed were a result of Photos own doing and not as a result of the limited down time of
the printer. See, White and Summers, Uniform Commercial Code § 12-10 c. (4th Ed. 1995).

14
Question No. 2: Grading Guidelines

1. Business Organizations – Promoter liability and piercing the corporate veil.

Comments: The candidates should analyze Diane’s potential liability under the lease as a
promoter and a shareholder (applying piercing the corporate veil principles).

7 points

2. Sales – Limitation of remedy to repair or replacement of defective parts

Comments: The candidates should discuss the ability of a seller to limit remedies to an exclusive
remedy or remedies and the limitations on a seller’s ability to do so under the Uniform
Commercial Code; i.e., does the remedy fail of its essential purpose or is the remedy
unconscionable.

7 points

3. Sales – Limitation of consequential damages is a commercial contract

Comments: The candidates should discuss the ability of a seller in a commercial contract to limit
consequential damages, and in particular, lost profits, and the limitation on the ability to do so,
i.e., unconscionability.

6 points

15
Question No. 3

Ted and Kayla were married in 1988 and have owned and continuously lived in a home

located in C County, Pa. Their one daughter, Mackenzie, turned 17 years of age on June 15,

2017. Unbeknownst to Ted, Kayla has allowed Mackenzie to have several beer parties at their

home while Ted was away on business. These parties have been attended by teenagers who have

consumed alcohol to the point of intoxication and Kayla, who has been present and provided the

beer for all of the parties, has allowed the teenagers to drive home after the parties.

On Saturday afternoon, June 17, 2017, while Ted was away on business, Kayla purchased

two kegs of beer from a beer distributor in C County by using her credit card. Kayla put the

purchase receipt in her pocket. The receipt had the following information on it: distributor name,

date and time of purchase, type and amount of beer purchased, the cost of the purchase, and

Kayla’s credit card number. Kayla proceeded home and had several of Mackenzie’s teenage

friends carry the kegs down to her basement where they iced them down for that evening’s party.

That evening, about 40 of Mackenzie’s teenage friends came over and consumed beer for

several hours. Some of the friends went upstairs and talked with Kayla while drinking their beer.

At midnight, John, Frank, Heidi, and Chrissy all left in the same car with Frank driving. Frank,

who spoke with Kayla before leaving and was visibly intoxicated, drove about two miles before

he failed to negotiate a curve in the road and hit a tree. Frank was killed instantly and the

remaining passengers were seriously injured but remained conscious. When the police arrived at

the accident scene, Heidi told them that they were at a party at Mackenzie’s house and that her

mother, Kayla, had bought the beer for the party.

After determining Kayla’s address, the police immediately went to her home. Kayla told

the police that she does not permit underage drinking at her home. Based upon the above

16
information given by Heidi, the police lawfully placed Kayla under arrest and charged her with

furnishing alcohol to minors and for involuntary manslaughter for Frank’s death. Kayla was

taken to the police station and, as part of the established booking process, her clothing was

searched resulting in the recovery of the receipt for the beer purchase on June 17, 2017. The

morning after the accident, Ted learned that Kayla had hosted the party and had done so in the

past. Ted went to the police station, where Kayla was being held, and he told Kayla he wanted a

divorce and had moved out. Kayla responded she would never agree to a divorce. Subsequent

investigation revealed that Frank was legally intoxicated at the time of the accident based on the

beer he consumed at Kayla’s home and that his intoxication caused the accident.

1. Do the facts support a charge of involuntary manslaughter against Kayla for


Frank’s death?

Assume Kayla proceeds to trial on the charges filed. At trial, the Commonwealth, after

having timely informed the defense before trial of its intention to do so, attempts to admit into

evidence that Kayla hosted several underage drinking parties prior to June 17, 2017, to establish

that she knew there was underage drinking taking place at her home. The defense objects to this

evidence on the basis that these prior incidents should not be admitted to prove that Kayla was

responsible for Frank’s death.

2. Aside from any habit argument, what argument(s) should the Commonwealth
make to support the admission of this evidence?

3. If Kayla’s attorney challenges the search and seizure that resulted in the police
recovering the beer purchase receipt on the basis that the police lacked a warrant,
how should the Commonwealth respond and how would the court likely rule?

4. Assume that Kayla is found guilty of both charges on July 7, 2018, and is
immediately sentenced to incarceration for a minimum sentence of 3 years.
Based on all the facts presented, what grounds for divorce does Ted have as of
July 7, 2018, to end his marriage to Kayla?

17
Question No. 3: Examiner’s Analysis

1. The facts do support a charge of involuntary manslaughter against Kayla for Frank’s
death due to Kayla’s reckless conduct.

“A person is guilty of involuntary manslaughter when as a direct result of the doing of an


unlawful act in a reckless or grossly negligent manner, or the doing of a lawful act in a reckless
or grossly negligent manner, he causes the death of another person.” 18 Pa.C.S.A. § 2504 (a).
“[I]nvoluntary manslaughter requires (1) a mental state of either recklessness or gross negligence
and (2) a causal nexus between the conduct of the accused and the death of the victim.”
Commonwealth v. McCloskey 835 A.2d 801, 806 (Pa. Super. 2003). The McCloskey court went
on to explain the following:

[R]ecklessness is statutorily defined as “consciously disregarding a substantial


and unjustifiable risk that the material element exists or will result from his
conduct. The risk must be of such a nature and degree that, considering the nature
and intent of the actor’s conduct and the circumstances known to him, its
disregard involves a gross deviation from the standard of conduct that a
reasonable person would observe in the actor’s situation.”

McCloskey, 835 A.2d at 806 quoting 18 Pa.C.S.A. § 302 (b)(3).

The Pennsylvania Supreme Court has construed the terms “reckless” and “grossly negligent”
as defining the equivalent state of mind for purposes of the involuntary manslaughter provision.
Commonwealth v. Huggins, 836 A.2d 862, 868 (2002) citing Commonwealth v. Comer, 552 Pa.
527, 716 A.2d 593, 597 (Pa. 1998) (definition of “recklessly” set forth in Section 302
“encompasses” gross negligence set forth in Section 2504) (citing Commonwealth v. Lobiondo,
501 Pa. 599, 462 A.2d 662 (Pa. 1983)).

In McCloskey, the court was faced with facts similar to the current case, in that
McCloskey (the mother) permitted her underage daughter to host an underage drinking party and
assisted in the planning of and preparation for the party. The mother was home for the entire
party and witnessed several teenagers drinking beer. The court concluded:

[A] parent who knows alcohol is being served to minors in her home is acting
recklessly when she allows the conduct to continue. That knowledge not only
constitutes a “gross deviation from the standard of conduct that a reasonable
person would observe in “her” situation,” 18 Pa. C.S.A. § 302 (b)(3), it also
constitutes a clear violation of the law. 18 Pa. C.S.A. § 6310.1(a).

McCloskey, 835 A.2d at 807.

The McCloskey court, quoting Commonwealth v. Nicotra, 625 A.2d 1259, 1264 (Pa.
Super. 1993), pointed out that the courts of this Commonwealth have held that “‘so long as the
defendant’s conduct started the chain of causation which led to the victim’s death, criminal

18
responsibility for the crime of homicide may properly be found.’” McCloskey, 835 A.2d at 808.
The court went on to conclude that when the mother furnished alcohol to the minor victim, she
started the chain of causation that led to the death of all of the victims. McCloskey, 835 A.2d at
808.

As applied here, Kayla purchased two kegs of beer for a party which she knew was going
to be attended by multiple teenagers. Kayla had several of the teenagers carry the beer down to
her basement and ice them down. Kayla had full knowledge that the teenagers were drinking as
the facts indicate that some of them were drinking in her presence. She also observed that Frank
was visibly intoxicated before he left her premises. This conduct would likely support the
mental state of recklessness or gross negligence for involuntary manslaughter as Kayla
consciously disregarded the substantial and unjustifiable risk that her conduct might lead to the
injury or death of another when she allowed Frank to drink and then drive in a visibly intoxicated
state. This would constitute a gross deviation from the standard of conduct that a reasonable
person should have observed in her situation.

The facts also indicate that the investigation into the accident revealed that Frank was
legally intoxicated at the time of the accident having consumed all of his alcohol at Kayla’s
home and that his intoxication caused the accident. Because Kayla permitted the minors,
including Frank, to consume alcohol and allowed Frank to drive, she started the chain of
causation which led to Frank’s death. Therefore, it is likely that there was a sufficient causal
nexus between Kayla’s conduct and Frank’s death to support a charge of involuntary
manslaughter.

In summary, it appears that the charge of involuntary manslaughter would be supported


by the facts presented.

2. The Commonwealth should argue that Kayla’s hosting of prior underage drinking
parties would be admissible under Pa.R.E. 404(b), otherwise known as prior bad act
evidence, and that the probative value of this evidence outweighs its prejudicial
effect.

The McCloskey court has stated the following:

The Pennsylvania Rules of Evidence govern the admission of prior bad acts
evidence. The relevant rule provides that “evidence of other crimes, wrongs or
acts is not admissible to prove the character of a person in order to show action in
conformity therewith” but may be admitted for “other purposes, such as proof of
motive, opportunity, intent, preparation, plan, knowledge, identity or absence of
mistake or accident.” Pa. R.E. 404 (b)(1) and (2). Admission is proper only if the
probative value of the evidence outweighs its potential for prejudice. Pa. R.E.
404 (b)(3).

McCloskey, 835 A.2d at 809-10, see also, Commonwealth v. Paddy, 800 A.2d 294 (Pa. 2002),
(holding that evidence of a defendant’s prior criminal activity is inadmissible to demonstrate bad
character or criminal propensity but may be admissible for various limited purposes provided

19
that its probative value outweighs the prejudicial effect likely to result from its admission and an
appropriate limiting instruction is given).

In a criminal case the prosecutor must provide reasonable notice in advance of trial, or
during trial if the court excuses pre-trial notice on good cause shown, of the general nature of any
such evidence the prosecutor intends to introduce at trial. Pa. R.E. 404(b)(3).

In McCloskey, the court was faced with the situation where teens had regularly met at the
home of a mother who hosted underage drinking parties and the mother was aware that the teens
were drinking alcohol. The court found that this evidence was offered to show the mother’s
knowledge that her residence was routinely used by minors as a place to drink alcohol. The
McCloskey court found that “the evidence . . . was probative of her awareness or knowledge of
the unlawful behavior and, as a result, was relevant to establish that she acted recklessly.” Id. at
810. It was also determined that the “trial court recognized the prejudicial nature of this
evidence and assessed its probative value against its prejudicial effect.” The Superior Court held
that the evidence was properly admitted and that the trial court did not abuse its discretion. Id.

As applied here, the facts indicate that the Commonwealth timely informed the defense
before trial of its intention to use the proposed bad act evidence. The facts further indicate that
Kayla hosted several underage drinking parties before the June 17, 2017, party that resulted in
Frank’s death. This evidence would be probative of her awareness or knowledge that teens were
regularly drinking alcohol at her home, which would be relevant to establish that she acted
recklessly under the circumstances. The Commonwealth should be prepared to argue that any
prejudice which might result from the admission of this evidence would be outweighed by the
probative value of the evidence to establish the element of recklessness for involuntary
manslaughter. Although this would ultimately be something that would have to be weighed by
the court, the Commonwealth would have persuasive arguments to admit this prior bad act
evidence on the facts presented. If the court agrees with the Commonwealth’s argument an
appropriate limiting instruction would need to be given to the jury.

3. The Commonwealth should respond that the receipt was discovered as a result of a
properly executed inventory search that is a recognized exception to the warrant
requirement; and the court should conclude that this was a legal search and that the
receipt is admissible.

“Under both the Fourth Amendment of the United States Constitution and Article I,
Section 8 of the Pennsylvania Constitution, a search conducted . . . without a warrant is deemed
to be per se unreasonable.” Commonwealth v. Cleckley, 738 A.2d 427, 429 (Pa. 1999) citing
Commonwealth v. Williams, 692 A.2d 1031 (Pa. 1997). The Pennsylvania Supreme Court stated
the following:

[I]nventory searches are a well-defined exception to the warrant requirement of the


Fourth Amendment and are a recognized part of our law: . . . “it is reasonable for police
to search the personal effects of a person under lawful arrest as part of the routine
administrative procedure at a police station house incident to booking and jailing the
suspect. The justification for such searches does not rest on probable cause, and hence

20
the absence of a warrant is immaterial to the reasonableness of the search. Indeed, we
have previously established that the inventory search constitutes a well-defined exception
to the warrant requirement.”

Commonwealth v. Nace, 571 A.2d 1389, 1391 (Pa. 1990) quoting South Dakota v. Opperman,
428 U.S. 364(1976).

According to the Nace court there are four goals that underlie such searches:

First, they protect the defendant’s property while he is in custody; second, police
are protected against theft claims when defendants are given their property upon
release; third, they serve to protect the police from physical harm due to hidden
weapons; and fourth, when necessary they ascertain or verify the identity of the
defendant.

Nace, 571 A.2d at 1391.

As applied here, Kayla was lawfully placed under arrest and was taken to the police
station. The facts indicate that as part of the established booking process her clothing was
searched resulting in the recovery of the receipt for the beer she purchased on June 17, 2017. A
search warrant would normally be required for the search of Kayla’s person. However, because
this search was part of a routine administrative procedure at the police station incident to her
booking, it would likely fall under the inventory search exception to the warrant requirement.
Because this receipt was properly obtained pursuant to the inventory search exception to the
search warrant requirement, it is likely that the court would rule that the receipt is admissible.

4. Ted likely has grounds for a fault-based divorce since Kayla has been sentenced to
imprisonment for a term of more than two years and Ted also has grounds for a no-
fault divorce based on irretrievable breakdown of the marriage as the couple have
been separated for more than one year.

The Domestic Relations Code, 23 Pa. C.S.A. § 3301(a)(5), provides that a “court may
grant a divorce to the innocent and injured spouse whenever it is judged that the other spouse has
. . . [b]een sentenced to imprisonment for a term of two or more years upon conviction of having
committed a crime.” As applied here, Ted is the innocent spouse. He was innocent of assisting
Kayla in her activities of hosting teenage drinking parties. See Murphy v. Murphy, 205 A.2d 647
(Pa. Super. 1964). Also, the facts indicate that Kayla has been convicted of the crimes charged
and has received a minimum sentence of three years of incarceration. Since this exceeds the
minimum two year imprisonment requirement under section 3301(a)(5) required to secure a
fault-based divorce, it is likely Ted would be able to secure a divorce from Kayla.

Alternatively, Ted could pursue a divorce under 23 Pa. C.S.A. § 3301(d)(1) that provides
that a “court may grant a divorce where a complaint has been filed alleging that the marriage is
irretrievably broken and an affidavit has been filed alleging that the parties have lived separate
and apart for a period of at least one year . . . .” Irretrievable breakdown is defined as
estrangement due to marital difficulties with no reasonable prospect of reconciliation. 23 Pa.

21
C.S.A. § 3103. Separate and apart is defined as cessation of cohabitation, whether living in the
same residence or not. In the event a complaint in divorce is filed and served, it shall be
presumed that the parties commenced to live separate and apart not later than the date that the
complaint was served. 23 Pa. C.S.A. § 3103. The period of separation commences when one
spouse clearly communicates his intent to dissolve the marital relationship to the other spouse.
Sinha v. Sinha, 526 A.2d 765, 767 (Pa. 1987). Since Ted explicitly notified Kayla on June 18,
2017, that he wanted a divorce, and had moved out, and they have been living separate and apart
for greater than one year as of July 7, 2018, it is likely that Ted could also pursue a no fault
ground for divorce under section 3301(d) under the Domestic Relations Code. 1

1
It is unlikely that Ted would be able to obtain a divorce based upon mutual consent under 23 Pa. C.S.A. § 3301(c)
as the facts expressly state that Kayla informed Ted that she would never agree to a divorce.

22
Question No. 3: Grading Guidelines

1. Criminal Law.

The applicants should discuss the elements of involuntary manslaughter and conclude that
the facts support the charge.

5 Points

2. Evidence

The applicants should identify prior bad act evidence and weigh its admission under the
probative value versus the prejudicial effect test in determining the admissibility of the
evidence.

5 Points

3. Criminal Procedure

The applicants should identify the issue as an inventory search exception to the search
warrant requirement and conclude that the purchase receipt would be admissible at trial.

6 Points

4. Family Law

The applicants should discuss whether Ted has grounds for a no-fault divorce under 3301(d)
based upon Ted and Kayla’s 1 year separation, and whether Ted has grounds for a fault
ground for divorce under 3301(a)(5) due to Kayla’s sentence of incarceration for a term in
excess of 2 years.

4 Points

23
Question No. 4

ManupPower (“MP”) employs 300 people and provides unarmed staffing for events all over the

country, such as concerts, sporting events, and political rallies. MP event personnel sell tickets the day of

events, collect tickets from attendees, provide seating information and directions, and monitor the

entrance and exit of attendees during events to identify “seat jumpers” trying to sit in non-assigned seats.

Ninety-five percent of the events staffed by MP also have an armed public police presence. Pursuant to

written policy, MP event personnel may not pursue or detain anyone and must contact public law

enforcement in the event of any physical aggression or resistance from event attendees.

In 2013, and on a whim, MP’s fitness-obsessed CEO implemented a physical fitness test that he

created from his own daily workouts as part of the final step of the hiring process for MP event personnel.

After interviews and background checks, candidates for event personnel positions were required to

complete various calisthenics and a one-mile run within a specified time period (the “Fitness Test”). If an

applicant passed the Fitness Test, that applicant was guaranteed an offer of employment as MP event

personnel. However, if an applicant failed the Fitness Test, then that applicant was not offered the job.

MP’s stated reason for using the Fitness Test in hiring decisions was so that MP clients could be

confident they were hiring “strong, tough, qualified event personnel.”

Between 2013 and the present, 400 applicants for security jobs made it to the final round of the

hiring process and took the Fitness Test. Half of those 400 applicants were men and half were women.

During those years, 90% of the 200 men administered the Fitness Test passed it (180 men total) and were

offered employment. However, only 5% of the 200 women who took the Fitness Test during those years

passed it (10 women total) and were offered employment. Betsy Smith, Libby Umber, and Liz Ellis

(collectively, “Plaintiffs”) all applied for MP security personnel jobs in 2016. Plaintiffs all excelled in

their interviews and passed their background checks. However, they failed the Fitness Test. Therefore,

none of the Plaintiffs was offered a position at MP.

After being denied employment at MP, Plaintiffs retained Big Law, a national plaintiffs’

employment law firm that has litigated hundreds of class action employment discrimination lawsuits.

24
Plaintiffs have exhausted their administrative remedies, and they have properly filed a lawsuit in the

appropriate federal district court against MP for alleged violations of Title VII of the Civil Rights Act of

1964 (the “Lawsuit”). Plaintiffs all want to reapply for jobs with MP in the future. Plaintiffs’ complaint

seeks declaratory and injunctive relief. Specifically, Plaintiffs want the court to declare that MP’s use of

the Fitness Test in making hiring decisions unlawful under Title VII and enjoin MP from using the

Fitness Test in connection with hiring decisions in the future. It is undisputed that there are at least 190

women who have failed the Fitness Test and not received offers of employment since MP starting using

this practice in making hiring decisions.

1. Plaintiffs’ complaint does not allege that MP engaged in intentional discrimination or that
the Fitness Test is facially discriminatory. Rather, Plaintiffs allege that MP’s practice of
using the Fitness Test as part of its hiring procedures results in an outcome that violates
Title VII. Based only on the facts set forth above, what theory of Title VII liability
should Plaintiffs assert against MP and with what likelihood of success?

2. Plaintiffs request that the federal district court certify a class of litigants comprised of “all
female applicants from the year 2013 to the present who were not offered MP event
personnel jobs based on their failure to pass the Fitness Test.” Plaintiffs seek to be the
class representative. What prerequisites will Plaintiffs have to establish to obtain class
certification and with what likelihood of success?

3. Assume for this question only that Plaintiffs did not seek class certification and
proceeded with only their individual claims. Over the past 16 months, MP has defended
the legality of the Fitness Test and vigorously litigated the Lawsuit, conducting countless
depositions, filing numerous motions, and producing thousands of pages of written
discovery. After the close of discovery and a month prior to the summary-judgment
deadline, Big Law received correspondence from MP’s attorney notifying Plaintiffs that
(1) effectively immediately, MP has stopped using the Fitness Test in making hiring
decisions, and (2) MP intends to seek dismissal of the Lawsuit because Plaintiffs have
requested only declaratory and injunctive relief related to the Fitness Test, and MP is no
longer using the Fitness Test to make hiring decisions. What federal constitutional
doctrine should MP raise in seeking dismissal of the Lawsuit and with what likely result?

25
Question No. 4: Examiner’s Analysis

1. Plaintiffs should assert a disparate impact claim against MP and are likely to succeed

Title VII of the Civil Rights Act of 1964 prohibits employment discrimination based on race,
color, religion, sex or national origin. 42 U.S.C. § 2000e-2(a)(1). “Title VII prohibits both intentional
discrimination (known as ‘disparate treatment’) as well as, in some cases, practices that are not
intended to discriminate but in fact have a disproportionately adverse effect on minorities
(known as ‘disparate impact’).” Ricci v. DeStefano, 557 U.S. 557, 577 (2009). In Griggs v.
Duke Power Co., 401 U.S. 424, 431 (1971), the Supreme Court first interpreted Title VII to
prohibit the use of facially neutral practices that are “discriminatory in operation.” Since Griggs,
Title VII has been amended to codify the relevant burdens of proof in disparate impact cases:

An unlawful employment practice based on disparate impact is established . . . if . . . a


complaining party demonstrates that a respondent uses a particular employment practice
that causes a disparate impact on the basis of [a class protected by Title VII] and the
respondent fails to demonstrate that the challenged practice is job related for the position
in question and consistent with business necessity . . . .

42 U.S.C. § 2000e-2 (k)(1)(A)(i) (emphasis added). If an employer demonstrates that a


challenged practice is “job related for the position in question and consistent with business
necessity,” a plaintiff may still prevail in a Title VII disparate impact case “by showing that the
employer refuses to adopt an available alternative employment practice that has less disparate
impact and serves the employer’s legitimate needs.” Ricci, 557 U.S. at 578 (citing 42 U.S.C. §
2000e-2 (k)(1)(A)(i), (ii)).

Disparate Impact

[T]he plaintiff [in a disparate impact case] must offer statistical evidence of a kind and
degree sufficient to show that the practice in question has caused the exclusion of
applicants for jobs or promotions because of their membership in a protected group. Our
formulations, which have never been framed in terms of any rigid mathematical formula,
have consistently stressed that statistical disparities must be sufficiently substantial that
they raise such an inference of causation.

Watson v. Fort Worth Bank & Trust, 487 U.S. 977, 994-95 (1988) superseded by statute on other
grounds, 42 U.S.C. 2000e-2(k)(1)(A).

The Equal Employment Opportunity Commission (EEOC) has issued guidelines stating
that “[a] selection rate for any race, sex, or ethnic group which is greater than four-fifths (4/5) (or
eighty percent) of the rate for the group with the highest rate will generally be regarded . . . as
evidence of adverse impact . . . .” 29 C.F.R. 1607.4(D). However, this is not a bright line
evidentiary rule. The regulation itself states that both smaller differences in selections rates may,
under some circumstances, establish disparate impact, and that greater differences may not in
other circumstances. Id. The Supreme Court has framed the question of the disparate impact
inquiry in terms such as whether challenged practices “operated to disqualify [a protected class]

26
at a substantially higher rate” than a non-protected class; Griggs, 401 U.S. at 426; whether tests
selected applicants “in a racial pattern significantly different from that of the pool of
applicants[,]” Albemarle Paper Co. v. Moody, 422 U.S. 405, 425 (1975); whether the employer
utilized “hiring and promotion practices disqualifying substantially disproportionate numbers of
blacks[,]” Washington v. Davis, 426 U.S. 229, 246-247 (1976); or whether there are standards
that “select applicants for hire in a significantly discriminatory pattern.” Dothard v. Rawlinson,
433 U.S. 321, 329 (1977).

Whether applying the EEOC 4/5 rule or the standards set forth in Supreme Court
precedent, Plaintiffs here can establish that the Fitness Test results in a disparate impact on
female applicants. Again, the selection rate for men under the MP Fitness Test is 90%, while the
selection rate for women is 5%. Because 90% of men in the final pool of applicants were hired,
72% of women (or 80% of 90%) from the same size pool must be hired in order to satisfy the 4/5
rule.1 However, only 5%, or 10 women, passed the Fitness Test and received offers of
employment. 5% is only 4.5% of 90%.2 This is woefully short of the 80% necessary to satisfy
the 4/5 rule. Moreover, the huge disparity between the number of men and women hired solely
because of performance on the Fitness Test evidences that the challenged practice (use of the
Fitness Test) “operated to disqualify [women] at a substantially higher rate” than it did men.
Griggs, 401 U.S. at 426. Thus, Plaintiffs here can easily establish their initial burden of showing
that the challenged practice – use of the Fitness Test as a job prerequisite – has a disparate
impact on women.

Business Necessity

Because the Fitness Test has a disparate impact on women, the burden will shift to MP to
show that the Fitness Test is “job related for the position in question and consistent with business
necessity.” 42 U.S.C. § 2000e-2(k)(1)(A)(i). In Griggs, the Supreme Court held that a
defendant could overcome a disparate impact claim by showing that employment tests that have
a discriminatory effect “bear a demonstrable relationship to successful performance of the jobs
for which it was used” and that “any given requirement must have a manifest relationship to the
employment in question.” Id. at 431, 432. In Albemarle Paper Co. v. Moody, 422 U.S. 405
(1975), the Supreme Court adopted EEOC guidance and held that test results must predict or
correlate with “important elements of work behavior which comprise or are relevant to the job or
jobs for which candidates are being evaluated.” 422 U.S. at 431 (quoting 29 C.F.R. § 1607.4(c)).
The Dothard Court stated that discriminatory employment requirements must “be shown to be
necessary to safe and efficient job performance.” Dothard, 433 U.S. at 331 n.14. In Dothard,
the employer argued that height and weight requirements for prison guards that had a
discriminatory impact on women were a business necessity because strength is an essential
quality for corrections officers. The Court noted that the employer had failed to identify the
specific amount of strength necessary to perform the job and had not demonstrated a relationship
between strength and its height and weight requirements. Id. at 331-32.

1
90 x .8 = 72.
2
90 x .05 = 4.5.

27
In all of the business necessity defense cases discussed above, the Supreme Court
required some evidence of the challenged test as it related to job performance. Employers must
provide evidence that the challenged practice “measure[s] the person for the job and not the
person in the abstract.” Dothard, 433 U.S. at 332 (quotation omitted). See also, EEOC v. Dial
Corp., 469 F.3d 735, 742 (8th Cir. 2006) (stating that a validity study3 of employment tests “is
not necessary if the employer demonstrates the procedure is sufficiently related to safe and
efficient job performance”); Lanning v. Southeastern Pa. Transp. Auth., 181 F.3d 478, 490 (3d
Cir. 1999) (asking whether a cutoff score on physical fitness test for transit officers measures
“the minimum qualifications necessary for the successful performance of the job in question”).

Here, there are no facts to indicate that the Fitness Test given to event personnel job
applicants at MP are in any way related to predicted job performance. MP’s only proffered
reason for administering the Fitness Test is so that MP’s customers will know they are getting
“strong, tough, qualified event personnel.” Yet, the actual job duties do not require MP event
personnel to be “strong” or “tough.” Essentially, these employees are ticket-takers and ushers at
events. In fact, if an event attendee becomes physically aggressive or resistant, MP event
personnel are prohibited from detaining or pursuing such attendees and are required to contact
law enforcement. Thus, there seems no reason to test candidates’ physical strength or
cardiovascular endurance, and MP is unlikely to be successful in asserting the “business
necessity” defense to Plaintiffs’ disparate impact claim.

Less Discriminatory Alternative

Typically, if an employer can establish the business necessity defense, a plaintiff may
still succeed in a disparate impact case where he or she can establish the existence of a less
discriminatory alternative device. If, however, an employer fails to demonstrate that the
challenged practice or screening device is a business necessity, the plaintiff is not “required to
show the absence of a nondiscriminatory alternative.” Dial Corp., 469 F.3d at 743. As set forth
above, there are no facts whatsoever to support the contention that the Fitness Test is “job related
for the position in question and a business necessity.” Therefore, Plaintiffs will not have the
burden of establishing the existence of a less discriminatory alternative to the Fitness Test.

Because Plaintiffs will be able to present statistical evidence that the Fitness Test
disproportionately affects female applicants, and MP will be unable to satisfy its burden to show
that the Fitness Test is job related for the position in question and consistent with a business
necessity, Plaintiffs are likely to be successful in their disparate impact claim under Title VII.

2. The district court will apply the requirements of federal rule 23 and likely certify
the class.

A class action lawsuit is “‘an exception to the usual rule that litigation is conducted by
and on behalf of the individual named parties only.’” Wal-Mart Stores, Inc. v. Dukes, 564 U.S.
338, 348 (2011) (quoting Califano v. Yamasaki, 442 U. S. 682, 700-01 (1979)). Federal Rule of

3
In 1978, the EEOC adopted Uniform Guidelines on Employee Selection Procedures (“UGESP”), providing
employers with guidance on how to determine if a test or selection procedure is lawful when analyzing alleged
disparate impact claims under Title VII. 29 C.F.R. Part 1607.

28
Civil Procedure 23(a) sets forth four requirements - commonly referred to as numerosity,
commonality, typicality, and adequate representation – designed to ensure that a named plaintiff
is an appropriate class representative. Id. at 349 (noting that the prerequisites of Federal Rule
23(a) “effectively limit the class claims to those fairly encompassed by the named plaintiff’s
claims”) (quotation and citation omitted)).

Specifically, Federal Rule 23(a) provides:

(a) Prerequisites. One or more members of a class may sue or be sued as


representative parties on behalf of all members only if:

(1) the class is so numerous that joinder of all members is impracticable;

(2) there are questions of law or fact common to the class;

(3) the claims or defenses of the representative parties are typical of the
claims or defenses of the class; and

(4) the representative parties will fairly and adequately protect the interests of the
class.

F.R.C.P. 23(a) (emphasis added).

In addition to satisfying all four requirements of Federal Rule 23(a), a putative class
representative must also establish that the putative class meets at least one of the three criteria of
Federal Rule 23(b). Wal-Mart, 564 U.S. at 345. Federal Rule 23(b) provides as follows:

(b) Types of Class Actions. A class action may be maintained if Rule 23(a) is satisfied
and if:

(1) prosecuting separate actions by or against individual class members would


create a risk of:

(A) inconsistent or varying adjudications with respect to individual


class members that would establish incompatible standards
of conduct for the party opposing the class; or

(B) adjudications with respect to individual class members that, as a


practical matter, would be dispositive of the interests of the
other members not parties to the individual adjudications or
would substantially impair or impede their ability to protect
their interests;

(2) the party opposing the class has acted or refused to act on grounds that

29
apply generally to the class, so that final injunctive relief or
corresponding declaratory relief is appropriate respecting the class
as a whole; or

(3) the court finds that the questions of law or fact common to the class
members predominate over any questions affecting only individual
members . . . .

F.R.C.P. 23(b).

Numerosity

Here, the representative plaintiffs seek certification of a class of approximately 190


women who have been denied employment at MP because they did not pass the Fitness Test
(200 applicants less the 5%, or 10 women, who were offered jobs). It is almost certain that the
court will hold the proposed class-size is sufficient to meet the numerosity requirement set forth
in Federal Rule 23(a)(1). “No minimum number of plaintiffs is required to maintain a suit as a
class action, but generally if the named plaintiff demonstrates that the potential number of
plaintiffs exceeds 40, the first prong of Rule 23(a) has been met.” Stewart v. Abraham, 275 F.3d
220, 226-27 (3d Cir. 2001) (citing 5 James Wm. Moore et al., Moore’s Federal Practice §
23.22[3][a] (Matthew Bender 3d ed. 1999)).

Commonality and Typicality

The requirements of commonality and typicality both “seek to assure that the action can
be practically and efficiently maintained and that the interests of the absentees will be fairly and
adequately represented.” Baby Neal for and by Kanter v. Casey, 43 F.3d 48, 56 (3d Cir. 1994).
“[C]ommonality requires the plaintiff to demonstrate that the class members ‘have suffered the
same injury.’” Wal-Mart, 564 U.S. at 349-50 (quoting General Telephone Co. of the Southwest
v. Falcon, 457 U.S. 147, 157 (1982). Stated another way, for Rule 23(a)(2) to be satisfied,
claims must “depend upon a common contention [that is] . . . of such a nature that it is capable of
classwide resolution – which means that determination of its truth or falsity will resolve an issue
that is central to the validity of each one of the claims in one stroke.” Id. at 350. In Wal-Mart, a
plurality of the Supreme Court held that there was no commonality of claims within a putative
class of 1.5 million female employees of Wal-Mart because the challenged pay and promotion
decisions were the result of discretionary decisions made by local supervisors rather than arising
out of a company-wide policy or practice. Id. at 352-60.

“The typicality inquiry is intended to assess whether the action can be efficiently
maintained as a class and whether the named plaintiffs have incentives that align with those of
absent class members so as to assure that the absentees' interests will be fairly represented.”
Baby Neal, 43 F.3d at 57 (citing 3B JAMES W. MOORE & John E. Kennedy, MOORE’S FEDERAL
PRACTICE ¶ 23.06-02 (1993); 1 H. NEWBURG & A. Conte, NEWBERG ON CLASS ACTIONS § 3.13
(1992)). The purpose of the typicality requirement is “to preclude certification of those cases
where the legal theories of the named plaintiffs potentially conflict with those of the absentees . .
. .” Id. (citation omitted).

30
While commonality and typicality are separate requirements for class certification, the
Supreme Court merged them together in the disparate impact context in holding the District
Court erred in certifying a class because the class representative failed to satisfy the commonality
and typicality requirements of Rule 23(a)(1) and (a)(2) in a Title VII action. Falcon, 457 U.S. at
158. In Falcon, the Court stated a “wide gap” existed between an individual’s claim of
discrimination in a promotion decision and the existence of a class of individuals who share the
same injury as that individual. Id. at 157. The Court explained that specific questions of law or
fact “common to the claims of [the class representative] and of the members of the class he
sought to represent” were necessary for a court to certify a class. Id. at 158. To that end, the
Falcon court indicated that “a class action on behalf of every applicant or employee who might
have been prejudiced by [a biased testing procedure] clearly would satisfy the commonality
and typicality requirements of Rule 23(a).” Id. at 159 n. 15 (emphasis added).

At issue here is MP’s use of the Fitness Test to determine whether to offer event
personnel jobs to applicants. Every member of the putative class applied for an MP event
personnel position, successfully interviewed, and passed a background check. However, each
was also denied an offer of employment because she did not pass the Fitness Test. This scenario
is clearly different than those in Wal-Mart and Falcon where the Supreme Court did not find
commonality to be present. Rather, it involves a biased testing procedure applied evenly across
all candidates for the job. As the Supreme Court said in Falcon, this “clearly would satisfy the
commonality and typicality requirements of Rule 23(a).” Id.

Adequate Representation

Finally, Plaintiffs must adequately represent the other members of the putative class.
“The adequacy inquiry under Rule 23(a)(4) serves to uncover conflicts of interest between
named parties and the class they seek to represent.” Amchem Prods., Inc. v. Windsor, 521 U.S.
591, 625 (1997). To that end, a class representative must have the same interest and suffer the
same injury as the class members she seeks to represent. Id. at 626 (citations and quotations
omitted). “Adequate representation depends on two factors: (a) the plaintiff’s attorney must be
qualified, experienced, and generally able to conduct the proposed litigation, and (b) the plaintiff
must not have interests antagonistic to those of the class.” New Directions Treatment Servs. v.
City of Reading, 490 F.3d 293, 313 (3d Cir. 2007) (quotation and citation omitted). See also, In
re Drexel Burnham Lambert Group, 960 F.2d 285, 291 (2d Cir. 1992).

Here, all of the members of the putative class are identically situated – they were denied
an offer of employment at MP because of their inability to pass the Fitness Test. Thus, there is
no basis on which the court could conclude that the named plaintiffs have interests antagonistic
to the class members they seek to represent. Moreover, Plaintiffs have retained a national
plaintiffs’ employment law firm that has successfully certified classes in hundreds of
employment discrimination cases. There are no facts that would suggest that plaintiff’s counsel
is not “qualified, experienced, and generally able to conduct the proposed litigation.” Id.

Thus, Plaintiffs would likely be able to meet all of the requirements for class certification
set forth in Rule 23(a).

31
Rule 23(b)

Once the requirements of Federal Rule 23(a) are determined to have been met, the court
must examine whether at least one of the criteria in Rule 23(b) is satisfied. F.R.C.P. 23(b). Here,
the facts support a finding that Plaintiffs can satisfy Rule 23(b)(2), which requires that: “the
party opposing the class has acted or refused to act on grounds that apply generally to the class,
so that final injunctive relief or corresponding declaratory relief is appropriate respecting the
class as a whole[.]” F.R.C.P. 23(b)(2).

Here, Plaintiffs’ allegations are that MP (the party opposing the class) improperly utilized
the Fitness Test in deciding whether to make offers of employment (i.e., acted or refused to act),
and that every member of the putative class was denied an offer of employment as a result (i.e.,
grounds that apply to the class). Plaintiffs are requesting identical injunctive relief that is
appropriate to the class as whole (i.e., an injunction against utilizing the Fitness Test in future
hiring decisions).

Because Plaintiffs can meet all of the requirements of Federal Rule 23(a) and the criteria
set forth in Rule 23(b)(2), it is likely that the district court will certify a class of all women
denied offers of employment as MP event personnel in the past four years based on their inability
to complete the Fitness Test.

3. MP should argue that plaintiffs’ claims are moot and plaintiffs should respond that
voluntary cessation of allegedly unlawful conduct is insufficient to moot their claims.

If MP is intent on trying to have the Lawsuit dismissed at this late stage of the litigation,
it should raise the doctrine of mootness, arguing that there is no longer a live “case or
controversy” because MP is no longer administering or using the Fitness Test in connection with
hiring decisions.

Article III of the United States Constitution limits the judicial power of federal courts to
“cases or controversies.” Friends of the Earth, Inc. v. Laidlaw Envtl. Servs., Inc., 528 U.S. 167,
180 (2000) (“The Constitution’s case-or-controversy limitation on federal judicial authority, Art.
III, § 2, underpins both our standing and our mootness jurisprudence . . .”). “‘Simply stated, a
case is moot when the issues presented are no longer ‘live’ or the parties lack a legally
cognizable interest in the outcome.’” Los Angeles Cty. v. Davis, 440 U.S. 625, 631 (1979)
(quoting Powell v. McCormack, 395 U.S. 486, 496 (1969)).

At first glance, MP’s decision to stop utilizing the Fitness Test would indicate that its
claims are moot because Plaintiffs’ complaint requests only declaratory and injunctive relief
related to MP’s use of the test in hiring decisions. However, Plaintiffs should respond to any
effort of MP to get the case dismissed on the basis of mootness by raising one of the “long-
recognized exceptions” to the doctrine, specifically, the exception of “voluntary cessation” of an
allegedly unlawful practice. Laidlaw, 528 U.S. at 189-90.

It is well settled that a defendant’s voluntary cessation of a


challenged practice does not deprive a federal court of its power to

32
determine the legality of the practice. If it did, the courts would be
compelled to leave the defendant . . . free to return to his old ways.
In accordance with this principle, the standard we have announced
for determining whether a case has been mooted by the defendant’s
voluntary conduct is stringent: A case might become moot if
subsequent events made it absolutely clear that the allegedly
wrongful behavior could not reasonably be expected to recur. The
heavy burden of persuading the court that the challenged conduct
cannot reasonably be expected to start up again lies with the party
asserting mootness.

Id. (citations and quotations omitted).

One factor that the Supreme Court has looked at to determine whether voluntary
cessation renders a claim moot or is likely to start again in the future is whether or not the
defendant has defended its actions. Parents Involved in Cmty. Schs. v. Seattle Sch. Dist. No. 1,
551 U.S. 701, 719 (2007) (rejecting a school district’s mootness argument where the district
ceased utilizing a challenged race-based policy during the course of litigation, in part because
“the district vigorously defends the constitutionality of its race-based program, and nowhere
suggests that if this litigation is resolved it its favor it will not resume using race to assign
students.”); see also, DeJohn v. Temple Univ., 537 F.3d 301, 304, 310-11 (3rd Cir. 2008)
(rejecting mootness argument where the defendant university modified a challenged policy three
weeks prior to the dispositive motion deadline but had defended its policy throughout litigation;
did not change it until discovery was almost over; and there was no indication that the university
had deliberated over its decision in such a way to indicate that it would not “revert back to its old
policy” in the future).

As in the above cases, there are no facts to suggest that MP would not return to using the
Fitness Test in connection with hiring decisions in the future. MP vigorously defended the
legality of the Fitness Test for more than a year and litigated the case extensively. It was only at
the eleventh hour, a few weeks before summary judgment motions were due, that MP ceased the
challenged practice. Simultaneously, MP indicated that it was going to seek dismissal of the
case.

In sum, there are no facts upon which the court would conclude that MP will not restart
the challenged practice. Accordingly, the court is unlikely to dismiss the Lawsuit.

33
Question No. 4: Grading Guidelines

1. Employment Law

Applicants should demonstrate knowledge of the relative burdens of proof for disparate impact claims
under Title VII and apply a set of facts to these elements to reach a well-reasoned conclusion.

8 points

2. Civil Procedure

Applicants should demonstrate knowledge of the prerequisites for class certification contained in Federal
Rule of Civil Procedure 23 and apply a set of facts to these criteria to reach a well-reasoned conclusion.

8 points

3. Constitutional Law

Applicants should identify mootness as a possible basis for dismissal under the facts presented and exhibit
knowledge of the voluntary cessation exception to the mootness doctrine to reach a well-reasoned
conclusion.

4 points

34
Question No. 5

Owner subdivided his farmland near Blueberry Hill, Pennsylvania, into two large parcels,

Lot A and Lot B, pursuant to a recorded subdivision plan (“the Plan”). The Plan included the

following declaration (the “Declaration”): “Alcoholic beverages shall not be made, sold, or

consumed on any parcels in this Plan. The benefits and burdens of this restriction shall be

applicable to all grantees, their heirs, successors, and assigns.” Owner eventually conveyed Lots

A and B to separate grantees by valid deeds that expressly referred to the Declaration. After

several succeeding valid transfers over an extended period of years, Lot A came to be owned by

KBL, a national cable television sports channel, which used the building previously constructed

on Lot A as its broadcast studios. Lot B came to be owned by Gatebriar, which operated an in-

patient drug and alcohol treatment facility on the property. The deed conveying Lot A to KBL

did not mention the Declaration. The deed conveying Lot B to Gatebriar stated that the

conveyance was “subject to all restrictions of record.”

KBL hired Al, who lived in Big City, Pennsylvania, for its most prestigious on-air job:

doing play-by-play for KBL’s nationally broadcast Sunday night pro-football games. Al’s

contract with KBL was for two years at $750,000 per year terminable only for cause. After

signing his KBL contract, Al became engaged to marry Brenda, his long-time agent, who

handled every aspect of Al’s broadcast career. Before marrying Al, Brenda conveyed Blackacre,

her Big City home, to her brothers, Chuck and Dave, using a deed form that she found on the

Internet. The deed stated in part: “Said Grantor does grant, bargain, sell, alien and enfeoff

Blackacre JOINTLY unto the said Grantees and their heirs forever.”

Following their marriage, Al and Brenda purchased a home in Blueberry Hill known as

Whiteacre. The deed validly conveying Whiteacre simply described Al and Brenda as the

35
grantees, and not as husband and wife.

Prior to the start of the second year of Al’s broadcasting contract, KBL materially

breached by firing Al without cause. Despite her best efforts, Brenda only was able to obtain an

offer from a low-watt Blueberry Hill radio station to have Al do play-by-play for local high

school football games for $25,000. Al, who also participated in the search for a new job,

rejected this offer and spent his time playing on-line “Fantasy Football.” In the process, Al ran

up a large personal debt on his Big Bank credit card that he was unable to pay.

KBL contacted What-A-Mess (“Mess”), a local contractor, about remodeling one of its

studios. Mess stated, “I’ll do the job for $200,000. You have 30 days to respond.” KBL’s

authorized representative replied, “Our budget is a little tight right now. We’ll keep your offer

under advisement, but if you wish to close the matter at once we’ll pay you $125,000.” Mess did

not respond to KBL’s statement.

1. Before the end of the 30 days, KBL’s representative called Mess and stated, “We
found some extra money in our budget. KBL accepts your offer to remodel its
studio sets for $200,000.” Was a valid contract formed between KBL and Mess?

2 (a). One year after Brenda conveyed Blackacre to Chuck and Dave, Chuck suddenly
died. By will, Chuck left his entire estate to the Human Fund, a Big City charity.
What is the state of title to Blackacre following Chuck’s death?

(b). When Al did not pay his huge credit card bill, Big Bank sued and obtained a
judgment solely against Al. To recover on this judgment, Big Bank wants to
force a sale of Whiteacre. Will Big Bank succeed in forcing a sale of Whiteacre?

3. Al sued KBL for breach of contract. What contract law argument should KBL
raise to preclude or reduce Al’s damages for material breach of his contract and
with what result?

4. Assume for purposes of this question only that Mess and KBL entered into a new
contract to convert part of KBL’s studios into a sports bar/brew pub called
“KBL’s Out.” What relief should Gatebriar seek to stop the construction of the
sports bar/brew pub on Lot A and what legal theory, if any, supports its right to
relief?

36
Question No. 5: Examiner’s Analysis

1. A valid contract was created between KBL and Mess because Mess’s offer was not
rejected by KBL’s counter-offer. KBL’s response clearly manifested an intention
that its counter-offer should not be understood as a rejection of Mess’s offer.

To create a contract under Pennsylvania law, “there must be an offer on one side and an
acceptance on the other side.” Cawthorne v. Erie Ins. Grp., 782 A.2d 1037, 1038 (Pa. Super.
2001).

The Restatement (Second) of Contracts describes an offer as “a manifestation of


willingness to enter into a bargain, so made as to justify another person in understanding that his
assent to that bargain is invited and will conclude it.” RESTATEMENT (SECOND) OF CONTRACTS
§ 24 (Am. Law Inst. 1981); accord, Philadelphia Newspapers, Inc. v. Unemployment Comp. Bd.
of Review, 426 A.2d 1289, 1290 n.3 (Pa. Cmwlth. 1981). “It is basic contract law that one
cannot suppose, believe, suspect, imagine or hope that an offer has been made. An offer must be
intentional, definite, [sic] in its terms and communicated, otherwise the minds cannot meet. . . .
An offer must define its terms, specify the thing offered and be an intention of the present or the
future to be bound.” Morosetti v. Louisiana Land and Exploration Co., 564 A.2d 151, 152 (Pa.
1989) (footnotes omitted). “Whether a statement is intended as an offer must be examined in
light of the surrounding circumstances.” Reed v. Pittsburgh Bd. of Pub. Educ., 862 A.2d 131,
135 (Pa. Cmwlth. 2004). Some of the circumstances relevant to the determination of whether an
offer is made “‘include the terms of any previous inquiry, the completeness of the terms of the
suggested bargain, and the number of persons to whom a communication is addressed.’” Beaver
Valley Alloy Foundry, Co. v. Therma–Fab, Inc., 814 A.2d 217, 222 (Pa. Super. 2002), quoting
RESTATEMENT (SECOND) OF CONTRACTS, § 26 cmt. c.

In this case, Mess stated, “I’ll do the job for $200,000.” Mess’s statement clearly
manifested an intent to be bound under definite terms and would be viewed as an offer to KBL.

“An offer creates a power of acceptance in a specified offeree to transform the offeror’s
promise into a contractual obligation.” Philadelphia Newspapers, Inc., 426 A.2d at 1290 n.3. “It
is a basic principle of the law of contracts that an acceptance must be unconditional and
absolute.” O’Brien v. Nationwide Mut. Ins. Co., 689 A.2d 254, 258 (Pa. Super. 1997) (quotation
and citation omitted). However, “a reply to an offer which purports to accept it, but changes the
conditions of the offer, is not an acceptance, but is a counter-offer.” First Home Sav. Bank, FSB
v. Nernberg, 648 A.2d 9, 15 (Pa. Super. 1994), appeal denied, 657 A.2d 491 (Pa. 1995) (citations
omitted).

Section 39(1) of the Restatement (Second) of Contracts defines a counter-offer as “an


offer made by an offeree to his offeror relating to the same matter as the original offer and
proposing a substitute bargain differing from that proposed by the original offer.”
RESTATEMENT (SECOND) OF CONTRACTS § 39(1) (1981). A counter-offer normally has the
same effect as a rejection of an offer because it terminates the offeree’s power to accept the
original offer. First Home Savings Bank, 648 A.2d at 15; see also, RESTATEMENT (SECOND) OF

37
CONTRACTS § 39, cmt. a. (“It is often said that a counter-offer is a rejection, and it does have the
same effect in terminating the offeree's power of acceptance.”). KBL’s reply that “if you wish to
close the matter at once we’ll pay you $125,000” proposed a substitute bargain and would be
viewed as a counter-offer to Mess.

The normal operation of a counter-offer as a rejection of the original offer, however, can
be overcome by a clear manifestation of “a contrary intention” by an offeree. RESTATEMENT
(SECOND) OF CONTRACTS § 39(2). Although KBL’s authorized representative made a counter-
offer by proposing a new price for Mess to remodel the KBL studios, KBL also stated that it was
taking Mess’s original offer “under advisement.” By using this language, the offeree (KBL)
clearly manifested an intention to the offeror (Mess) that the counter-offer should not be
understood as a rejection of Mess’s original offer. Id., cmt. c; see also, id. § 38 (2) (“A
manifestation of intention not to accept an offer is a rejection unless the offeree manifests an
intention to take it under further advisement.”). Therefore, even though KBL made a counter-
offer, it did not reject Mess’s offer. Further, each party had a power of acceptance: Mess to
accept KBL’s counter-offer at $125,000 and KBL to accept Mess’s original offer at $200,000.
See, JOHN E. MURRAY, JR., MURRAY ON CONTRACTS, § 43[D] (5th ed. 2011).

By clearly manifesting an intention that its counter-offer to Mess should not be


understood as a rejection of Mess’s original offer, KBL still retained its power to accept Mess’s
offer within the specified time period.1 Because KBL accepted Mess’s offer to remodel the KBL
studios at a cost of $200,000 within the thirty-day period specified in the offer, a valid contract
was formed between KBL and Mess. See, RESTATEMENT (SECOND) OF CONTRACTS § 39(2),
Illustration 3.

2 (a). The language in Brenda’s deed will be construed as creating a tenancy in common
between Chuck and Dave. Upon Chuck’s death, Dave and the Human Fund each
will own undivided one-half interests in Blackacre as tenants in common.

Concurrent or joint estates are estates owned by two or more persons at the same time.
RALPH E. BOYER ET AL, THE LAW OF PROPERTY: AN INTRODUCTORY SURVEY, 100 (West
Publishing Co. 4th ed. 1991). There are three types of joint estates: tenancy in common, joint
tenancy, and tenancy by the entireties. JAN Z. KRASNOWIECKI, KRASNOWIECKI ON REAL
PROPERTY LAW AND PRACTICE, 137 (PBI Press 2nd ed. 2008). Of the three types of joint or
concurrent ownership of property, tenancy by the entireties can exist only between co-tenants
who are married. Id. at 156; Boyer, supra, at 103.

To create a joint tenancy, the so-called four “unities” of time, title, interest and
possession must be present. In re Estate of Quick, 905 A.2d 471, 474 (Pa.2006) (citation
omitted). The distinctive characteristic of a joint tenancy is the right of survivorship. Upon the
death of one joint tenant, the surviving joint tenant becomes the sole owner of the entire interest

1
As the master of the offer, the offeror may place whatever time limitations upon the exercise of the power to accept
the offer as the offeror deems desirable. MURRAY, supra § 42 (B). “If an offer prescribes the place, time or manner
of acceptance its terms in this respect must be complied with in order to create a contract.” RESTATEMENT
(SECOND) OF CONTRACTS § 60 (1981). See also, Id. § 41 (1), cmt. a. (An offer “may prescribe a time limit for
acceptance.”)

38
in the property. In re Parkhurst’s Estate, 167 A.2d 476, 478 (Pa. 1961). “The survivorship
characteristic of a joint tenancy precludes a joint tenant from disposing of his interest by will.”
General Credit Co. v. Cleck, 609 A.2d 553, 556 (Pa. Super. 1992), appeal discontinued, 613
A.2d 560 (Pa. 1992).

In contrast, “[a] tenancy in common is [a concurrent] estate in which there is unity of


possession but separate and distinct titles.” In re Sale of Property of Dalessio, 657 A.2d 1386,
1387 n.1 (Pa. Cmwlth. 1995). Unlike a joint tenancy, where joint tenants own an undivided part
of the whole, each tenant in common owns the whole of the undivided interest. 1 RONALD M.
FRIEDMAN, LADNER PENNSYLVANIA REAL ESTATE LAW, § 8.03 (6th ed. 2013). “It is basic
property law that ‘a right of survivorship’ is not associated with a ‘tenancy in common.’”
Margarite v. Ewald, 381 A.2d 480, 481 (Pa. Super. 1977). “In a tenancy in common, . . . when
one co-tenant dies, his interest descends or passes by will to his heirs or devisees; the remaining
co-tenants acquire no additional interest in such an estate.” Edel v. Edel, 424 A.2d 946, 948 (Pa.
Super. 1981) (citations omitted).

At early common law, joint tenancies were favored and a conveyance or devise to two or
more persons who were not husband and wife was presumed to create a joint tenancy with the
right of survivorship. Under contemporary law, joint tenancies with right of survivorship are in
disfavor and the presumption now is that all co-owners who are not husband and wife hold
jointly as tenants in common. In re Estate of Michael, 218 A.2d 338, 340-41 (Pa. 1966).
Pennsylvania law follows the trend of disfavoring joint tenancies with right of survivorship.
Pennsylvania Bank & Trust Co. v. Thompson, 247 A.2d 771 (Pa. 1968). By statute, the incident
of survivorship was taken away as a presumption and an instrument creating a joint estate is
presumed to create a tenancy in common. See, 68 P.S. § 110 (2004).

Nevertheless, Pennsylvania’s statute on joint tenancies has been held only to be a statute
of construction; it does not proscribe the creation of a joint tenancy with right of survivorship.
Teacher v. Kijurina, 76 A.2d 197, 201 (Pa. 1950). In light of Pennsylvania’s statutory
preference for tenancy in common, the question whether a particular conveyance or devise to
two or more persons creates a joint tenancy with right of survivorship becomes one of intent.
Maxwell v. Saylor, 58 A.2d 355, 356 (Pa.1948). In order to engraft the right of survivorship on a
co-tenancy which might otherwise be a tenancy in common, the intent to do so must be
expressed with sufficient clearness to overcome the statutory presumption that survivorship is
not intended. Isherwood v. Springs-First Nat’l Bank, 74 A.2d 89, 91 (Pa.1950).

In Kijurina, the Pennsylvania Supreme Court observed that the necessary intent to
overcome the statutory presumption against a joint tenancy with right of survivorship could be
discerned by the grantor’s use of words in the conveyance such as “as joint tenants and not as
tenants in common,” or “survivor,” or “with the right of survivorship.” Kijurina, 76 A.2d at 201.
(internal quotations and citations omitted). By contrast, in this case, the pre-printed language in
Brenda’s deed granting Blackacre to Chuck and Dave only used the word “jointly.” In the
absence of any language in the deed indicating an intention to create a joint tenancy with right of
survivorship, a court applying Pennsylvania law would probably hold that the statutory
presumption in favor of tenancy in common should prevail in this instance and that Chuck and
Dave owned Blackacre as tenants in common.

39
Because Chuck’s interest in Blackacre was owned as a tenant in common, he was able to
transfer that interest by will to the Human Fund. Therefore, Dave and the Human Fund each
own an undivided one-half interest in Blackacre as tenants in common.

2 (b). By virtue of their marital status, Al and Brenda own Whiteacre as tenants by the
entireties. Big Bank cannot recover its judgment against Al by forcing a sale of
Whiteacre as long as the tenancy by the entireties remains intact.

The deed conveying Whiteacre did not state that Al and Brenda were married. The
failure of the deed to note this fact, however, is inconsequential under Pennsylvania law because
the conveyance of real estate to two grantees who are husband and wife is presumed to create a
tenancy by the entireties. Holmes Estate, 200 A.2d 745, 747 (Pa.1964). It is the actual married
status of the grantees and not necessarily the words stated or omitted in the instrument that
determines their right to take as tenants by the entireties. 1 RONALD M. FRIEDMAN, LADNER
PENNSYLVANIA REAL ESTATE LAW, § 8.04 (b) (6th ed. 2013).

To overcome the presumption that a conveyance to grantees who are married creates a
tenancy by the entireties, there must be clear and convincing evidence of a contrary intent.
Holmes, 200 A.2d at 747, see also Brenner v. Sukenik, 189 A.2d 246, 249 (Pa.1963). In this
case, there are no facts that would indicate an intention to create a different estate in Whiteacre
between Al and Brenda. Therefore, the presumption would control and Al and Brenda would
own Blackacre as tenants by the entireties.

A tenancy by the entireties is predicated upon legal unity of husband and wife. Beihl v.
Martin, 84 A. 953, 954 (Pa.1912). From the inception of the estate, “each spouse is seized of the
whole or the entirety and not of a share, moiety, or divisible part.” Gasner v. Pierce, 134 A. 494,
495 (Pa. 1926). Because a tenancy by the entireties is grounded in the conception of the estate as
a single indivisible unit, the law in Pennsylvania is quite clear on whether a creditor of one of the
tenants can enforce a debt against property held as a tenancy by the entireties:

[A] judgment creditor may execute on entireties property to enforce his judgment
if both spouses are joint debtors. However, if only one spouse is a debtor,
entireties property is immune from process, petition, levy, execution or sale. In
the latter situation, the judgment creditor has only a potential lien against property
held by the entireties based on the debtor spouse’s expectancy to become sole
owner.

ISN Bank v. Rajaratham, 83 A.3d 170, 174 (Pa. Super. 2014) (emphasis added, quotation
and citations omitted).

As long as Al and Brenda are both alive and married to one another, they will continue to
own Whiteacre as tenants by the entireties. Therefore, Big Bank’s attempt to recover Al’s
personal credit card debt by forcing a sale of Whiteacre will not be successful.

40
3. KBL should argue that Al’s damages should be reduced because he failed to make
reasonable efforts to mitigate his damages. The success of KBL’s argument depends
on whether Al’s rejection of the job doing radio play-by-play for high school
football games is considered to be reasonable.

“[I]t is a universally accepted rule that [an injured] promisee cannot recover those
damages for breach of contract which he could have avoided through the exercise of reasonable
diligence if he can do so without incurring undue risk, expense or humiliation.” JOHN E.
MURRAY, JR., MURRAY ON CONTRACTS, § 123 [A] (5th ed. 2011); see also, RESTATEMENT
(SECOND) OF CONTRACTS § 350 (1) (1981). The rule of mitigation or limitation of avoidable
losses is often referred to in Pennsylvania case law as a “duty” to mitigate damages. See, e.g.,
TruServ Corp. v. Morgan's Tool & Supply Co., Inc., 39 A.3d 253, 262 (Pa. 2012) (citations
omitted). A failure to reasonably attempt to mitigate damages reduces a party’s recovery by the
amount of loss that could have been avoided by reasonable mitigation. State Pub. Sch. Bldg.
Auth. v. W.M. Anderson Co., 410 A.2d 1329, 1331 (Pa. Cmwlth. 1980); RESTATEMENT
(SECOND) OF CONTRACTS § 350, cmt. b.

In determining whether an injured party has acted appropriately to mitigate damages, the
test to be applied is one of reasonableness. Id., § 350 (2). Whether an injured party’s conduct in
response to a breach is reasonable is to be determined from all the facts and circumstances of
each case at the time in question. Schnabel Assoc., Inc. v. T & M Interiors, 507 A.2d 1241, 1243
(Pa. Super. 1986) citing Toyota Indus. Trucks U.S.A. Inc. v. Citizens Nat’l Bank of Evans City,
611 F.2d 465, 471 (3d Cir. 1979). The burden is on the breaching party to show how losses
could have been avoided through the efforts of the injured party. TruServ Corp., 39 A.3d at 262.

In a breach of employment contract case, “the measure of damages is the wages which
were to be paid less any amount actually earned or which might have been earned through the
exercise of reasonable diligence in seeking other similar employment.” Delliponti v. DeAngelis,
681 A.2d 1261, 1265 (Pa. 1996) (citation omitted). “An employee, however, need not accept
employment that is substantially different from the original employment to mitigate damages.”
MURRAY, supra § 123[D]. Thus, the burden is upon the breaching party to prove that other
substantially equivalent positions were available to the injured party and that the injured party
failed to use reasonable diligence in attempting to secure those positions. Delliponti, 681 A.2d at
1265.

Whether an injured party’s “substitute employment is ‘comparable,’ ‘substantially


similar,’ or ‘substantially equivalent’ to the original employment can be a difficult issue.”
MURRAY, supra § 123 [D]. In this case, Al had an offer to do play-by-play for high school
football games on a low-watt Blueberry Hill radio station for $25,000. Al’s original employment
was doing television play-by-play of KBL’s nationally broadcast professional football games, the
most prestigious on-air job with the network, at $750,000 per year. The trier of fact will have to
carefully consider all of these circumstances in determining whether the other employment
opportunity offered to Al was substantially similar to his previous position at KBL and thus
made his failure to take the new position with the Blueberry Hill radio station an unreasonable
effort to mitigate his damages due to KBL’s material breach of contract.

41
4. Gatebriar should seek injunctive relief to prevent KBL from constructing a sports
bar/brew pub on Lot A on the basis that the Declaration prohibiting alcoholic
beverages from being made, sold or consumed on any parcels in the Plan is
enforceable as an equitable servitude.

Covenants affecting real property are mainly of two kinds. A restriction which is binding
only upon the covenantor and which is not intended to be a continuing charge on real estate is
classified as a personal covenant. Those restrictions which are so closely connected with real
estate that their benefits and burdens pass to subsequent purchasers and assignees of the real
estate are classified as covenants running with the land. DeSanno v. Earle, 117 A. 200, 202 (Pa.
1922).

By convention and as a historical fact, covenants running with the land evolved into two
types. This first type, which developed in the English common law courts, is known as a
covenant running at law or a real covenant. ROGER A. CUNNINGHAM ET AL, THE LAW OF
PROPERTY, 468 (West Publishing Co. 1984). The remedy sought by a person seeking to enforce
a real covenant is money damages. Id. at 484. The second type of covenant running with the
land, which developed in the English Chancery Court, is known as an equitable servitude. Id. at
468. The remedy sought by a person seeking to enforce an equitable servitude is equitable in
nature. Id. at 497. Pennsylvania law provides that an action in equity seeking injunctive relief is
available to enforce a covenant running with the land contained in a deed or recorded plan.
Peters v. Davis, 231 A.2d 748, 752 (Pa.1967); Doylestown Twp. v. Teeling, 635 A.2d 657, 660
(Pa. Cmwlth. 1993), appeal denied, 653 A.2d 1234 (Pa. 1994). Since Gatebriar wants to stop the
construction of the sports bar/brew pub on Lot A, it would want equitable relief in the form of an
injunction. Therefore, Gatebriar must prove that the restriction prohibiting alcoholic beverages
from being made, sold, or consumed on any parcels in the Plan is enforceable as an equitable
servitude.

To qualify as an equitable servitude, the restriction concerning the use of the land must
satisfy several requirements. First, the restriction must touch and concern the land. This
requirement generally involves the imposition of some benefit or burden upon the land by some
restriction of its use. Goldberg v. Nicola, 178 A. 809, 810 (Pa.1935); WILLIAM B. STOEBUCK &
DALE A. WHITMAN, THE LAW OF PROPERTY, § 8.15 (West Publishing Co. 3d ed. 2000). In this
case, the restriction in the Declaration governing the use of the lots within the Plan touches and
concerns the land because it imposes a benefit or burden by prohibiting the manufacture, sale, or
consumption of alcoholic beverages on any parcels in the Plan.

The second requirement in determining whether a restriction concerning the land is


enforceable as an equitable servitude is the intent that the restriction run with the land. DeSanno,
117 A. at 202. “[T]o ascertain such intent, resort may be had to the words of the covenant read
in the light of the surroundings of the parties and the subject of the grant.” Id. Neither formal
nor specific technical language is needed to create an equitable servitude. See, Logston v.
Penndale, Inc., 576 A.2d 59, 62 (Pa. Super. 1990), citing Elec. City Land & Improvement Co. v.
West Ridge Coal Co., 41 A. 458, 462 (Pa. 1898). Nevertheless, the use of certain terms and
language normally indicates that a covenant running with the land is intended. In this case, the
restriction on the use of the two parcels in the Plan’s Declaration states, “The benefits and

42
burdens of this restriction shall be applicable to all grantees, their heirs, successors, and assigns.”
Language stating that a particular restriction is binding upon the grantees’ heirs and assigns is
“generally decisive of the question” whether a covenant is intended to run with the land. Leh v.
Burke, 331 A.2d 755, 760 (Pa. Super. 1974); RESTATEMENT (THIRD) OF PROPERTY
(SERVITUDES) § 2.2, cmt. d (2000).

The final requirement2 needed to enforce a covenant running with the land as an equitable
servitude is that the party against whom the covenant is to be enforced had notice of the
restriction. RALPH E. BOYER ET AL, THE LAW OF PROPERTY: AN INTRODUCTORY SURVEY, 323 (West
Publishing Co. 4th ed. 1991). In this case, KBL did not have actual notice of the Declaration
requiring that no alcoholic beverages be made, sold, or consumed on Lot A. Further, no
reference to the restriction was included in his deed to Lot A. KBL, however, cannot use either
the absence of actual notice of the restriction or the lack of any reference to the restriction in his
deed to avoid compliance with the restriction on the use of the property. Longstanding
Pennsylvania law provides not only that a property owner had the duty to become aware of the
recorded restrictions in his chain of title but also that the property owner would be bound by such
restrictions in the absence of actual notice. Finley v. Glenn, 154 A. 299, 301 (Pa. 1931); accord,
Doylestown Twp., 635 A.2d at 661; Leh, 331 A.2d at 761. Because the restriction here was in the
recorded Declaration, KBL will be deemed to have constructive notice of the restriction and the
restriction can be enforced against it.

In conclusion, Gatebriar should seek injunctive relief to prevent KBL from constructing
its sports bar/brew pub on Lot A because the restriction in the Declaration prohibiting alcoholic
beverages from being made, sold, or consumed on any parcels in the Plan is enforceable as a
common law equitable servitude.

2
Because an equitable servitude evolved under common law into an interest in land, some commentators include an
instrument which complies with the statute of frauds as another necessary element. RALPH E. BOYER ET AL, THE
LAW OF PROPERTY: AN INTRODUCTORY SURVEY, 325-26 (West Publishing Co. 4th ed. 1991). In this case, the
written and recorded Declaration would satisfy the writing requirement. Privity of estate also was a required
element under the common law for a covenant to run at law. “The traditional privity doctrine was grounded in the
notion that because covenants did not create property rights, they could run only if there was an ‘identification’
between the original covenantee or covenantor and the successor. Privity supplied that identification.”
RESTATEMENT (THIRD) OF PROPERTY, (SERVITUDES) 5 Intro. Note (2000). The traditional requirement of
privity has been discarded by the Restatement “because its conceptual basis has disappeared with the modern
recognition that covenants create property rights, it is not supported by any modern cases, and it produces unsound
results.” Id. Even if privity was a requirement under Pennsylvania law, a privity analysis would not be necessary in
this case because privity of estate is not required for a covenant to run in equity. BOYER, 323.

43
Question No. 5: Grading Guidelines

1. Contract Formation - Offer, Acceptance, and Counter-Offer

Comments: Candidates should define and discuss two of the basic elements necessary to form a
contract under the common law: offer and acceptance. Candidates should recognize that a reply
to an offer which changes the terms and conditions of the offer is not an acceptance, but a
counter-offer that normally terminates the power to accept the original offer. Candidates further
should recognize that an offeree’s manifestation of a contrary intent can overcome the normal
operation of a counter-offer as a rejection of the original offer and thereby allow the offeree to
accept the original offer.

5 Points

2. Concurrent Estates – Tenants in Common, Joint Tenancy with Right of


Survivorship and Tenancy by the Entireties

Comments: Candidates should discuss the differences between the concurrent estates of joint
tenancy with right of survivorship and tenancy in common and the present common law and
statutory presumption favoring tenancy in common. Candidates should analyze the particular
language in Brenda’s deed and reach a well-reasoned conclusion concerning whether the
presumption favoring tenancy in common was overcome in this case.

Candidates should discuss Pennsylvania law’s presumption that a conveyance to a husband and
wife creates a tenancy by the entireties even when the words of the instrument fail to state the
existence of the marriage. Candidates should recognize that an estate held by the entireties
cannot be attached or sold by a creditor of one of the tenants as long as both tenants are alive and
remain married. Candidates should apply these principles to the stated facts and reach the
conclusion that Big Bank’s attempt to force a sale of the entireties property recover on its
judgment against Al will not be successful.

7 Points

3. Damages for Breach of Contract –Mitigation of Damages

Comments: Candidates should recognize that a party injured by breach of contract is required to
mitigate damages and that a failure to reasonably attempt to mitigate damages reduces the
injured party’s recovery by the amount of avoidable losses. Based upon the stated facts,
candidates should apply the law in reaching a well-reasoned conclusion concerning whether Al
made reasonable efforts to reduce his damages by finding comparable employment.

3 Points

44
4. Enforcement of an Equitable Servitude

Comments: Candidates should recognize that the Declaration restricting the use of the lots in the
Plan is enforceable as an equitable servitude. Candidates shall identify the elements necessary
for an equitable servitude to benefit successive owners and to be binding upon successive owners
and apply the elements to the stated facts. Candidates should conclude their analysis by
discussing the specific relief to enforce the restriction.

5 Points

45
Question No. 6

On November 8, 2015, 25-year-old Penny was driving in C County, Pennsylvania, when her car

left the road during a storm and crashed. Her head hit the windshield. During Penny’s call to 911, she

stated, “This was all my fault. I shouldn’t have driven so fast on that slippery road.” When emergency

medical technicians (“EMTs”) arrived, Penny was not breathing. The EMTs resuscitated her and

transported her to the C County Hospital (“Hospital”).

At Hospital, an x-ray revealed a fracture with compression to the front of Penny’s skull. Dr. Stone

(“Stone”), a Hospital employee and an authorized member of its surgical staff, was on call. Stone was

new to Hospital, having just completed his residency. Hospital had hired Stone knowing he was very

inexperienced. After reviewing Penny’s x-rays and vital signs, Stone immediately performed emergency

surgery to stop the bleeding in Penny’s brain and relieve pressure in her skull. Penny was Stone’s first

unsupervised brain surgery. Hospital did not have any protocols in place requiring inexperienced surgeons

to consult with another surgeon before performing brain surgery. After repairing all visible sources of

bleeding, Stone noted a small amount of blood still seeping, but could not immediately see its source. He

closed without searching further for the origin of the bleeding, hoping it would stop on its own.

After surgery, Penny was placed in a medically induced coma and given medications to minimize

swelling in her brain; however, her condition worsened. Stone monitored Penny’s condition for 24 hours

before concluding that there was still some bleeding in Penny’s brain. In a second surgery the day after

the first, Stone found and repaired a small bleed in an artery. Penny then improved. On November 16,

2015, Penny was brought out of the coma. She was suffering significant, permanent memory loss. That

same day, Stone fully discussed with Penny and her parents all his actions and decisions in treating her,

and they took notes of the discussion.

46
Based on a medical opinion Penny’s parents obtained from another doctor on November 20, 2015,

Penny filed a civil complaint against Stone and Hospital in the C County Court of Common Pleas. Penny

alleged that Stone acted negligently and deviated from acceptable physician standards, either by missing a

damaged artery or by nicking a previously undamaged artery during the first surgery. Based on the

medical opinion, Penny claimed her memory loss resulted from the continued bleeding in her brain after

the first surgery.

1. Penny asserted negligence claims against Hospital based on both direct and vicarious
liability relating to Stone’s acts or omissions. Assuming Penny’s claim was properly filed,
how would the court analyze each of these two claims?

2. Assume, for this question only, that Penny filed her negligence lawsuit on November 30,
2017. If the defense properly seeks dismissal of Penny’s claim asserting that the claim was
untimely, how would the court likely rule?

3. Assume, for this question only, that after Penny files her lawsuit, Hospital implements a
new protocol requiring inexperienced surgeons to consult with an experienced surgeon
before performing brain surgery. Penny wants to introduce evidence of the new protocol at
trial. Assume this evidence is otherwise relevant and not subject to hearsay objections.
Under Pennsylvania rules of evidence, if Penny offers this evidence to demonstrate that the
former absence of such protocols constituted negligence by Hospital, how should the court
rule on admissibility of the evidence for that purpose?

4. Assume Penny’s statement to the 911 operator will be admissible at trial regarding Penny’s
negligence claims against Hospital and Stone. Assume the jury, having been properly
instructed, finds that Penny was negligent, along with Stone and Hospital, and all three
were at fault in causing her injury. Under Pennsylvania law:

(a) If the jury finds Stone and Hospital each 30% at fault and finds Penny 40% at fault for
her injuries, how does this affect the amount, if any, of the verdict Penny may recover?

(b) If the jury finds Stone and Hospital each 20% at fault and Penny 60% at fault for her
injuries, how does this affect the amount, if any, of the verdict Penny may recover?

47
Question No. 6: Examiner’s Analysis

1. Penny may assert a claim of direct liability for negligent hiring and supervision, as well as a
claim of vicarious liability against Hospital.

Direct negligence

“Pennsylvania recognizes the doctrine of corporate negligence as a basis for hospital liability
separate from the liability of the practitioners who actually have rendered medical care to patients.” Rauch
v. Mike-Mayer, 783 A.2d 815, 826 (Pa. Super. 2001). As with all negligence claims, a claim of negligence
against a hospital requires proof of all four elements of that claim: a duty of care, a breach of that duty,
resulting injury, and damages. Id. at 826-27. In Pennsylvania, a hospital’s specific duty of care to
emergency patients is governed by the Restatement (Second) of Torts § 323 (1965):

One who undertakes, gratuitously or for consideration, to render services to another which
he should recognize as necessary for the protection of the other’s person or things, is
subject to liability to the other for physical harm resulting from his failure to exercise
reasonable care to perform his undertaking, if

(a) his failure to exercise such care increases the risk of such harm, or

(b) the harm is suffered because of the other’s reliance upon the undertaking.

Thompson v. Nason Hosp., 527 Pa. 330, 340, 591 A.2d 703, 708 (1991) (quoting Riddle Mem’l Hosp. v.
Dohan, 504 Pa. 571, 576-77, 475 A.2d 1314, 1316 (1984) (quoting Restatement (Second) of Torts § 323
(1965))).

A hospital owes a non-delegable duty to ensure the patient’s safety and wellbeing while at the
hospital. “Therefore, an injured party does not have to rely on and establish the negligence of a third
party” in order to bring a direct claim against a hospital. Thompson, 527 Pa. at 339, 591 A.2d at 707.
Pertinent here, the Hospital’s duty to its patients includes “a duty to select and retain only competent
physicians . . . [and] a duty to [supervise] all persons who practice medicine within its walls[,]” as well as
“a duty to formulate, adopt and enforce adequate rules and policies to ensure quality care for the patients.”
Id. at 339-40, 591 A.2d at 707.

For a hospital to be liable to a patient for direct negligence, the plaintiff must “show that the
hospital had actual or constructive knowledge of the defect or procedures [that] created the harm[, and] the
hospital’s negligence must have been a substantial factor in bringing about the harm to the injured party.”
Id. at 341, 591 A.2d at 708. Thus, the plaintiff must show that the hospital’s negligence was a factual
cause in bringing about her injury. Gorman v. Costello, 929 A.2d 1208, 1212-13 (Pa. Super. 2007)
(quoting SSJI Civ 3.15).

[F]actual cause does not mean it is the only, primary, or even the most important factor in
causing the injury. A cause may be found to be a factual cause as long as it contributes to
the injury in a way that is not minimal or insignificant. To be a contributing factor, the

48
defendant’s conduct need not be the only factor. The fact that some other cause concurs
with the negligence of the defendant in producing an injury does not relieve the defendant
from liability as long as [his] [her] own negligence is a factual cause of the injury.

Id. at 1212 (internal quotations and citation omitted).

Applying this rule to the facts of the question, Hospital owed a duty of care to Penny to retain
competent surgeons, to oversee their activities, and to have proper protocols in place. To prove direct
liability against Hospital, Penny must show that Hospital breached its duty of care because Stone was not
competent, because it did not adequately supervise his surgical activities, or because it did not implement
protocols sufficient to assure adequate supervision or timely follow-up care after a patient’s condition
worsens post-surgery. Relevant to these issues is the fact that Hospital knew of Stone’s level of
inexperience, but did not put protocols in place that might have provided mandatory consultation and
supervision in connection with his performance of emergency brain surgery. The finder of fact would
consider these issues in deciding whether Hospital violated its duty of care.

Penny must also prove that Hospital’s conduct was a factual cause of her memory loss. However,
to plead a claim of direct liability against Hospital, Penny does not need to allege that Stone was negligent.

Vicarious liability

As a general rule, a master has the right to exercise control over the physical activities of the
servant within the time of service and is vicariously liable for the servant’s negligent acts committed
within the scope of his employment. Smalich v. Westfall, 440 Pa. 409, 415, 269 A.2d 476, 481 (1970)
(quoting Restatement (Second) of Agency § 219 (1958) (further citation omitted)). More specifically, a
hospital owes a duty to its patients and may be liable under agency theory, as the principal, for the
negligence of surgical personnel. Tonsic v. Wagner, 458 Pa. 246, 253, 329 A.2d 497, 501 (1974). See
also 40 P.S. § 1303.516 (A hospital may be vicariously liable for acts of a health care provider if a
reasonable person would justifiably believe the care in question was being rendered by the hospital or its
agents, or if the care was represented as being rendered by the hospital or its agents.).

To sustain a claim of vicarious liability against Hospital, Penny must satisfy the elements of a
negligence claim against Stone: a duty owed by Stone to Penny, a breach of that duty, and damages
directly resulting from the breach. Here, the facts do not indicate that Stone made any attempt to consult
any other surgeon or doctor, or to seek assistance with Penny’s surgery. Further, he concluded the surgery
and closed despite apparent ongoing bleeding in Penny’s brain.

As the facts indicate no dispute that Stone was an employee on duty in Hospital and was authorized
to perform surgery, a viable negligence claim against him will also constitute a viable claim of vicarious
liability against Hospital.

2. The court would likely rule that Penny’s complaint was filed outside the two-year statute of
limitations for a negligence claim and dismiss it.

The Pennsylvania Judicial Code (“Code”) (42 Pa.C.S. §§ 5501-5574) sets forth the time periods
within which an action must be filed. The Code specifically provides that a tort action must be

49
commenced within two years from the date the cause of action accrues. 42 Pa.C.S. § 5524 (2017). “Once
the prescribed statutory period has expired, the party is barred from bringing suit unless it is established
that an exception to the general rule applies which acts to toll the running of the statute.” Bickford v.
Johnson, 368 Pa. Super. 211, 220, 533 A.2d 1029, 1033 (1987). The running of the statute of limitations
in a tort case may be tolled during a period when, despite the exercise of reasonable diligence, the plaintiff
was unable to discover her injury or its cause. Id., 368 Pa. Super. at 215, 533 A.2d at 1031. In such an
instance, the cause of action accrues when the plaintiff reasonably should be aware of the injury and the
facts supporting a potential tort claim. See id.

Applying this statute to Penny’s claim, she was specifically made aware of all the events giving
rise to her claim when she was brought out of her coma, i.e., on November 16, 2015, and her cause of
action accrued at that time. Her memory loss would have been readily apparent. Further, Stone’s
discussion with her on that date explained all the facts concerning her surgeries, thus eliminating any
discovery rule issue. See id. Even if it can be argued that Penny did not know the cause of her injury on
November 16, 2015, at the latest she was aware on November, 20, 2015, when her parents got the second
opinion. Thus, the statute of limitations therefore ran on Penny’s claim, at the latest, on November 20,
2017. She filed her action on November 30, 2017, which means that her claim was filed outside the statute
of limitations. Consequently, under the facts assumed for this portion of the question, Penny’s claim will
be dismissed.

3. The court should rule that evidence of subsequent remedial measures is inadmissible to
suggest prior negligence.

The implementation of new protocols after Penny files her lawsuit, requiring that inexperienced
surgeons consult with an experienced surgeon before performing brain surgery, would be classified as a
subsequent remedial measure. See Pa.R.E. 407.

When measures are taken by a party that would have made an earlier injury or harm less
likely to occur, evidence of the subsequent measures is not admissible against that party to
prove:

 negligence;
 culpable conduct;
 a defect in a product or its design; or
 a need for a warning or instruction.

But the court may admit this evidence for another purpose such as impeachment or – if
disputed – proving ownership, control, or the feasibility of precautionary measures.

Pa.R.E. 407 (2013). Specifically, Pennsylvania law precludes admission of any evidence of the review
process or the resulting change in protocols, when offered for the purpose of imputing antecedent
negligence. See Larch v. Haverford State Hosp., 152 Pa. Cmwlth. 459, 465, 620 A.2d 37, 40-41 (1993).

The rationale behind this rule is twofold. First, because a negligence action depends on the
standard of care applicable at the time of the relevant events, subsequent repairs are not relevant; they may
have arisen from new or different circumstances, or from a change in the applicable duty of care. Second,

50
social policy encourages remedial measures that will enhance safety, and admitting evidence of remedial
measures against the party implementing them might improperly discourage that party from undertaking
such measures. Duchess v. Langston Corp., 564 Pa. 529, 535-36, 769 A.2d 1131, 1134 (2001).

Under Rule 407, however, if feasibility is disputed and the trial court deems the evidence relevant
to the question of whether the protocols were feasible for implementation prior to Penny’s surgery, the
evidence will be admissible for that purpose. Feasibility is generally determined based on considerations
such as cost, practicality, and technological possibility. Duchess, 564 Pa. at 554, 769 A.2d at 1146.
Duchess cautions that the exception for feasibility should be narrowly applied so that the exception does
not swallow the rule. Id.

Applying the rule to the facts of the question, the facts as given do not suggest any of the
exceptions listed in Rule 407. The implementation of new protocols would not be admissible merely for
the purpose of imputing antecedent negligence to the defendants.

4.a. If Penny is found contributorily negligent, she can still recover if she was less than 50%
responsible for her injury.

Contributory/comparative negligence is an affirmative defense that defendants can assert to lower


or eliminate the defendant’s liability. See Pa.R.C.P. No. 1030; 42 Pa. C.S. § 7102. If a plaintiff makes out
a prima facie case of negligence, the defendant has the burden of proof to show that the plaintiff was
contributorily negligent. Contributory negligence is shown by “disproving the use of due care on the part
of plaintiff by a preponderance of the evidence.” Smith v. Port Auth. Transit, 257 Pa. Super 66, 70-71, 390
A.2d 249, 251 (1978).

Pennsylvania’s comparative negligence statute provides, in pertinent part:

§ 7102. Comparative negligence.

(a) General rule. - - In all actions brought to recover damages for negligence resulting in death or
injury to person or property, the fact that the plaintiff may have been guilty of contributory
negligence shall not bar a recovery by the plaintiff or his legal representative where such
negligence was not greater than the causal negligence of the defendant or defendants against whom
recovery is sought, but any damages sustained by the plaintiff shall be diminished in proportion to
the amount of negligence attributed to the plaintiff.

42 Pa. C.S. § 7102(a) (2011).

Contributory negligence is neglect of the duty imposed upon a person to exercise ordinary care for
his or her own protection and safety that is a legally contributing cause of an injury. Trayer v. King, 241
Pa. Super. 86, 90, 359 A.2d 800, 802 (1976). The contributory negligence of a plaintiff, like the
negligence of a defendant, is lack of due care under the circumstances. Argo v. Goodstein, 438 Pa. 468,
480, 265 A.2d 783, 789 (1970). A plaintiff is contributorily negligent with respect to injuries which are
received as a result of a failure on her part to observe and avoid an obvious condition that ordinary care for
her safety would have disclosed. Skalos v. Higgins, 303 Pa. Super. 107, 114, 449 A.2d 601, 604 (1982).

51
Penny’s acknowledgment that she was driving too fast for the road conditions and was at fault for
the accident could support a defense of contributory negligence. Fact issues would remain for trial
concerning the cause or causes of Penny’s memory loss.

Case law has established that the party causing an injury and a physician subsequently aggravating
that injury or causing a new one are not joint tortfeasors because they act independently. See, e.g., Smith
v. Pulcinella, 440 Pa. Super. 525, 530, 656 A.2d 494, 497 (1995). However, that does not preclude
consideration of the parties’ relative fault under principles of comparative negligence. As one court
explained:

The critical issue . . . with respect to the comparative negligence and contributory
negligence questions is whether the negligence of various defendants and plaintiff were
substantial factors in causing the injuries. Where an automobile accident occurs, and an
injured party is then treated negligently by medical care personnel, one party might be
found causally negligent with respect to the accident and the medical personnel also be
found causally negligent in treatment of the injuries. The cause of action with respect to
both defendants might be considered in the same litigation and the jury would apportion
the negligence of the two tortfeasors according to the effect each had on the ultimate
injuries. Similarly, a defendant might negligently cause a “fender bender” accident
which results in injuries which are unexpectedly severe because the other driver was not
using a seat belt. The question in either case should be whether the negligence of the
given party was a substantial factor in causing the injury sustained by plaintiff; not
whether the negligence was a substantial factor in causing the vehicular impact.

Turner v. Scaife, 44 D. & C.3d 349, 354-55 (C.P. Lycoming 1987).

Here, the facts assume that a jury finds Penny partly at fault for her injury. Thus, the facts likewise
assume that Penny’s conduct and the defendants’ conduct joined in causing that injury.

“Pennsylvania[’s] Comparative Negligence Act replaced the harsh common law doctrine of
‘contributory negligence’ under which a plaintiff whose own negligence, however slight, contributed to
the happening of the accident in a proximate way, was barred from recovery.” Elder v. Orluck, 511 Pa.
402, 416, 515 A.2d 517, 524 (1986). Under the Act, a plaintiff can recover provided her fault is 50% or
less of the cause of her harm. The Elder court held that “[c]omparison of the plaintiff’s negligence to that
of the combined negligence of all defendants is consistent with and furthers the intent of the Act. It
insures that an injured plaintiff, who is otherwise entitled to a recovery, will not go uncompensated
because of the number of defendants who contributed to [her] injuries.” Id.

Applying the statute to the facts here, Penny was 40% at fault for her injury, while the combined
fault of the defendant’s was 60%. Accordingly, Penny will recover under this scenario, but her recovery
will be reduced by the percentage of her fault.

4.b. If the jury finds Penny was 60% responsible for her injury, she will not be able to recover for
her injury under the Comparative Negligence Act.

52
When a plaintiff’s negligence contributes more than 50% to her injury, the Act is a bar to her
recovery. 42 Pa. C.S. § 7102(a); See also Elder, 511 Pa. at 416, 515 A.2d at 524.

Under this scenario, Penny was 60% at fault for her injury, while the sum of the defendants’
percentage of fault was 40%. Thus, under the Comparative Negligence Act, Penny will recover nothing.

53
Question No. 6: Grading Guidelines

1. Torts – Professional Negligence and Vicarious Liability

Comments: The applicant should recognize that two claims are possible against the hospital – (1) direct
liability for negligent hiring and supervision, and (2) vicarious liability. The applicant should set forth the
elements of each cause of action and apply them to the facts of the question.

8 Points

2. Civil Procedure – Statute of Limitations

Comments: The applicant should recognize the statute of limitations issue and apply the two year statute
to the facts in the question and conclude that Penny’s complaint falls outside the statute.

3 Points

3. Evidence – Subsequent remedial measures

Comments: The applicant should recognize that Pennsylvania Rules of Evidence preclude admission of
evidence concerning subsequent remedial measures for the purpose of proving antecedent negligence, but
that the evidence may be admissible for another purpose, such as to establish feasibility.

4 points

4. Affirmative Defense – Comparative Negligence

Comments: The applicant should recognize that Pennsylvania’s comparative negligence statute governs
this affirmative defense and apply the defense to the facts in the relevant facts.

5 Points

54
PT
Question Number 3
PLACE BAR CODED APPLICANT LABEL HERE

on Examplify

Supreme Court of Pennsylvania


Pennsylvania Board of Law Examiners

Pennsylvania Bar Examination


July 2424
February andand
25,25,
2018
2015

PERFORMANCE TEST
July 24,24,
February 2018
2015

Use GRAY covered book for your answer to the Performance Test.
2018 Pa
© 2015 Board of Law Examiners
Pennsylvania Board of Law Examiners

55
Table of Contents

FILE
1. Assignment Memorandum ................................................................................................... 1

2. Plaintiff’s Complaint............................................................................................................ 2

3. Deed to Plaintiff’s Property ................................................................................................. 5

4. Deed to Defendant’s Property .............................................................................................. 6

5. Map of Parcels in Question .................................................................................................. 7

6. Formatting Memorandum for Drafting Preliminary Objections .......................................... 8

7. Sample Preliminary Objections ........................................................................................... 9

LIBRARY

1. Wolfe v. Porter, 592 A.2d 716 (Pa.Super. 1991) ................................................................. 10

2. Zeglin v. Gahagen, 812 A.2d 558 (Pa. 2002) ...................................................................... 13

56
FILE

57
Memorandum
TO: Applicants
FROM: Mark Kerwick, Managing Partner
RE: Assignment to Draft Preliminary Objections
DATE: February 27, 2018

Ten (10) days ago, our client, George Gilbert, was properly served with a complaint concerning the ownership
of a parcel of land near Lake Anne in Lake Anne Township, Luke County, Pennsylvania. The complaint was
filed in the Court of Common Pleas of Luke County, Pennsylvania. The plaintiff, Paul Kerrigan, claims
ownership of a tract of land measuring forty feet (40’) by fifty feet (50’), even though record title to the
property is in the name of George Gilbert.

The first part of your assignment is to draft two preliminary objections to the complaint asserting that the
complaint is legally insufficient (demurrer). The first preliminary objection in the nature of a demurrer is based
on Plaintiff’s failure to meet the elements of adverse possession. The second preliminary objection in the nature
of a demurrer is based on plaintiff’s failure to meet the elements of consentable/aquiescence in a boundary.
Although there are other preliminary objections that we could raise, your assignment is limited to the preceding
preliminary objections as I will be drafting any additional objections. Do not raise tacking or privity as part of
your preliminary objection based upon plaintiff’s failure to meet the elements of consentable/aquiescence in a
boundary. The second part of your assignment is to draft only the legal argument section of a brief in support of
the two preliminary objections. I will draft the remaining sections of the brief. Your legal argument section
should integrate relevant facts and set forth the law that will convince the court to grant the two preliminary
objections. Place the legal argument portion of the assignment at the end of your preliminary objections and
separate your legal argument with the heading “Legal Argument.”

Included in the attached File is the plaintiff’s complaint (attached to which are the deeds to the parties’
properties, and a map of the parcels in question), a formatting memorandum on drafting preliminary objections,
and a sample form of preliminary objection. Included in the attached Library is caselaw relevant to the two
objections. You should only use facts contained in the File, and you should only use the attached cases for your
preliminary objections and legal argument section. Do not rely upon your personal knowledge of these issues
or on cases and statutes not included in the Library. Instead you should base your objections, analysis, and
conclusions only upon the documents provided in the File and the Library.

Please set forth each preliminary objection and your legal argument under separate headings. In your legal
argument section, provide the applicable standards for granting preliminary objections and a reasoned legal
analysis supporting each preliminary objection. When citing to any authority, Bluebook citations are not
necessary; however, you must include sufficient informal citations to the appropriate authority, such that I will
know to which document you are referring.

58
PAUL KERRIGAN, Plaintiff : In the Court of Common Pleas of Luke County
:
v. : Civil Action - Law
: Action to Quiet Title
GEORGE GILBERT, Defendant : No. 2017 CIV 6241970

COMPLAINT

AND NOW, comes the Plaintiff, Paul Kerrigan, and in support of his Complaint avers as follows:

1. Plaintiff, Paul Kerrigan, is an adult competent individual who resides at Lot 3, Lake Anne

Township, Luke County, Pennsylvania.

2. Defendant, George Gilbert, is an adult competent individual who resides at 12 Church Street,

Lake Anne Township, Luke County, Pennsylvania.

3. Plaintiff is the titled owner of Lot 3 in the Lake Anne Subdivision, said property being more

particularly described in the Deed recorded with the Luke County, Pennsylvania, Recorder of Deeds Office at

instrument number 06-03-19952, and which is attached hereto as “Exhibit A.”

4. Lot 3 is a lakefront parcel measuring 40’ x 100’ that runs from the Lake on the front side of the

property to the public roadway on the back side of the property, with the front and back sides measuring 40’

each, and the sides of the property running from the lake to the road being 100’ each.

5. Defendant is the titled owner of Lot 6 in the Lake Anne Subdivision, said property being more

particularly described in the Deed recorded with the Luke County, Pennsylvania, Recorder of Deeds Office at

instrument number 09-04-20041, and which is attached hereto as “Exhibit B.”

6. Lot 6 is a large open field, consisting of approximately 50+ acres, with no fencing, trees or other

distinguishing characteristics.

7. Plaintiff and his predecessor in title have been the titled owners to Lot 3 for in excess of fifty-

three (53) years, with Plaintiff having purchased the parcel ten (10) years ago.

8. For no fewer than thirty (30) years, Plaintiff and his predecessor in title have used an area of land

40’ wide and 50’deep, which is located directly across the public road from the back side of Lot 3 and which is

encompassed as part of Lot 6. Plaintiff and his predecessor in title have been in actual, open, hostile,

59
continuous and exclusive possession of the 40’ x 50’ tract contained within Lot 6 for at least thirty (30) years.

A map of the Lake Anne Subdivision is attached hereto as “Exhibit C,” and Plaintiff’s counsel has drawn a

dotted line thereon depicting the 40’x50’ tract in question.

COUNT I – ADVERSE POSSESSION

9. Plaintiff incorporates each and every paragraph by reference as if more fully set forth herein.

10. Plaintiff and his predecessor in title have adversely possessed the 40’ x 50’ tract of land

referenced above for a period of time in excess of twenty-one (21) years.

11. Plaintiff and his predecessor in title have occupied and utilized said tract as their own, for their

own exclusive use, including parking of vehicles and conducting recreational activities. Plaintiff and his

predecessors have continuously exerted actual, open, hostile and exclusive possession of said tract for more

than twenty-one (21) years.

Wherefore, Plaintiff respectfully requests that the Court enter judgment in his favor and against

Defendant, declaring and adjudging that Plaintiff owns absolutely and is entitled to quiet and peaceful

possession of the 40’ x 50’ tract encompassed by the Deed to Lot 6, and which is located directly across the

public dirt roadway from Lot 3, and that the Recorder of Deeds be Ordered to execute a deed that can be

properly recorded to reflect that Plaintiff is the sole owner of the 40’ x 50’ tract of land.

COUNT II – CONSENTABLE/ACQUIESCENCE IN A BOUNDARY

12. Plaintiff incorporates each and every paragraph by reference as if more fully set forth herein.

13. Plaintiff and his predecessor in title have possessed the 40’ x 50’ tract referenced above for more

than thirty (30) years up through and including the present. Defendant has acquiesced to Plaintiff’s ownership

of the tract.

Wherefore, Plaintiff respectfully requests that the Court enter judgment in his favor and against

Defendant, declaring and adjudging that Plaintiff owns absolutely and is entitled to quiet and peaceful

possession of the 40’ x 50’ tract encompassed by the Deed to Lot 6, and which is located directly across the

60
public dirt roadway from Lot 3, and that the Recorder of Deeds be Ordered to execute a deed that can be

properly recorded to reflect that Plaintiff is the sole owner of the 40’ x 50’ tract of land.

Respectfully Submitted,

/s/ Atticus Mason


Atticus Mason, Esquire
1 Courthouse Square
Maintown, PA 99909
Counsel to Plaintiff

61
EXHIBIT A
Instrument No. 06-03-19952

THIS DEED

MADE THIS 1st day of February 2008,

Between Ethan Bishop, of the Town of Maintown, County of Luke, Commonwealth of Pennsylvania
(hereinafter “Grantor”), and

Paul Kerrigan, of the Town of Maintown, County of Luke, Commonwealth of Pennsylvania (hereinafter
“Grantee”),

WITNESSETH, that in consideration of One Hundred Fifty Thousand Dollars ($150,000), in hand paid,
the receipt whereof is hereby acknowledged; Grantor does hereby grant to Grantee, his Heirs and Assigns,

ALL that certain lot, piece or parcel of land situate, lying and being in the Township of Lake Anne,
County of Luke, and Commonwealth of Pennsylvania, bounded and described as follows, to wit:

All that certain piece or lot of land called and known as Lake Anne Subdivision Lot 3, in the Township
of Lake Anne, bounded and described as follows:

BEGINNING at a pin at which the high water mark of Lake Anne, Lot 3 and Lot 4 come together;
thence 100’ North to a pin where the public dirt road, Lot 3 and Lot 4 come together; thence 40’ West to a pin
where the public dirt road, Lot 3 and Lot 2 come together; thence 100’ South to a pin where the high water
mark of Lake Ann, Lot 2 and Lot 3 come together; thence 40’ East to the pin where the high water mark of
Lake Anne, Lot 3 and Lot 4 come together. Containing approximately four thousand (4,000) square feet.

BEING the same premise conveyed by Benjamin Maier to Ethan Bishop by Deed dated January 3, 1965,
and recorded in Luke County Deed Book 551, Page 400.

AND the said Grantor shall Warrant Generally the property hereby conveyed.

IN WITNESS WHEREOF, the Grantor has hereunto set his hand and seal the day and year first written
above.

WITNESS: /s/ Maria VonBergen GRANTOR: /s/ Ethan Bishop

62
EXHIBIT B
Instrument No. 09-04-20041

THIS DEED

MADE THIS 1st day of February 2008,

Between Lilly Townsley, of the Town of Maintown, County of Luke, Commonwealth of Pennsylvania
(hereinafter “Grantor”), and

George Gilbert, of the Town of Maintown, County of Luke, Commonwealth of Pennsylvania


(hereinafter “Grantee”),

WITNESSETH, that in consideration of Two Hundred Thousand Dollars ($200,000), in hand paid, the
receipt whereof is hereby acknowledged; Grantor does hereby grant to Grantee, his Heirs and Assigns,

ALL that certain lot, piece or parcel of land situate, lying and being in the Township of Lake Anne,
County of Luke, and Commonwealth of Pennsylvania, bounded and described as follows, to wit:

All that certain piece or lot of land called and known as Lake Anne Subdivision Lot 6, in the Township
of Lake Anne, bounded and described as follows:

BEGINNING at a pin at which the public dirt roadway, Lot 6 and Lot 9 come together; thence 1500’
North to a pin where the Lot 6, Lot 8, and Lot 9 come together; thence 1500’ West to a pin where Lot 6, Lot 8,
and Lot 7 come together; thence 1500’ South to a pin where the public dirt road, Lot 6 and Lot 7 come together;
thence 1500’ East to a pin where the public dirt roadway, Lot 6 and Lot 9 come together. Containing
approximately two million two hundred fifty thousand (2,250,000) square feet.

BEING the same premise conveyed by Dorothy Fendick to Lilly Townsley by Deed dated August 4,
1961, and recorded in Luke County Deed Book 301, Page 895.

AND the said Grantor shall Warrant Generally the property hereby conveyed.

IN WITNESS WHEREOF, the Grantor has hereunto set his hand and seal the day and year first written
above.

WITNESS: /s/ Gretchen Arcaro GRANTOR: /s/ Lilly Townsley

63
EXHIBIT C
Map of Lake Anne Subdivision (not to scale)

Lot 8

1500’

Lot 7 Lot 6 Lot 9

1500’

40’

Tract
In 50’
Question

Dirt Road

Lot 1 Lot 2 Lot 3 Lot 4 Lot 5

100’

40’

Lake Anne

64
Big Law Firm
TO: All Associates and Law Clerks
FROM: Mark Kerwick, Managing Partner
RE: Formatting Memorandum for Drafting Preliminary Objections
DATE: July 1, 2018

Use the following guidelines for drafting preliminary objections:

1. A preliminary objection is a pleading that sets forth a challenge to the sufficiency of a pleading,
often a complaint, based on the contents of that pleading. Preliminary objections to a complaint are
filed before a defendant answers the complaint, and are filed in an attempt to have the complaint, or
a portion thereof, dismissed before proceeding with further litigation.
2. Two or more preliminary objections may be raised in one pleading.
3. The preliminary objections shall be divided into paragraphs numbered consecutively.
4. Each paragraph in the preliminary objections shall contain as far as practicable only one material
allegation.
5. A material allegation is an assertion of fact that is essential to the objection.
6. The material facts on which a cause of action or defense is based shall be stated in a concise and
summary form.
7. Any part of a pleading may be incorporated by reference in another part of the same pleading or in
another pleading in the same action.
8. A party may incorporate by reference any matter which is recorded or transcribed verbatim in the
office of the prothonotary, clerk of any court of record, recorder of deeds or register of wills of such
county.
9. More than one preliminary objection may be raised in the same pleading; preliminary objections are
limited to following the grounds, as enumerated in Pennsylvania Rule of Civil Procedure No. 1028:
(1) lack of jurisdiction over the subject matter of the action or the person of the defendant,
improper venue or improper form or service of a writ of summons or a complaint;
(2) failure of a pleading to conform to law or rule of court or inclusion of scandalous or
impertinent matter;
(3) insufficient specificity in a pleading;
(4) legal insufficiency of a pleading (demurrer);
10. Each preliminary objection must be stated in a separate section of the pleading, and must have its
own heading (see sample preliminary objection attached hereto).
11. A paralegal will place a caption containing all relevant information at the top of your preliminary
objections. The paralegal will also place an appropriate notice to plead, verification, proposed order,
and signature block at the end of the preliminary objections. Thus, do not include any of these items
in the preliminary objections.
12. A sample preliminary objection to a complaint (also with no caption, notice to plead, verification, or
signature block) is attached hereto for your reference.

65
Sample Preliminary Objection

1. Plaintiff filed its complaint in the Court of Common Pleas of Luke County, Pennsylvania,
alleging that Plaintiff performed professional engineering services on Defendant’s behalf, and that Defendant
failed to pay Plaintiff for those services. See Plaintiff’s Complaint attached hereto as “Exhibit A.”

2. Plaintiff’s complaint admits that the project for which the services were allegedly performed was
a single family residence. See Exhibit A at paragraph 12.
3. Plaintiff’s complaint avers that this is the first and only project Defendant is working, and has
worked, on. See Exhibit A at paragraph 13.

Preliminary Objection One


Based Upon
Legal Insufficiency of Defendant’s Pleading
Under the Contractor and Subcontractor’s Payment Act

4. Defendant incorporates all averments of these preliminary objections herein by reference as if


more fully set forth at length.

5. Count IV of Plaintiff’s complaint seeks to recover against Defendant under the Contractor and
Subcontractor Payment Act, 73 P.S. § 501, et seq.

6. The Contractor and Subcontractor Payment Act is inapplicable to the instant matter.

7. The Contractor and Subcontractor Payment Act states "[t]his Act shall not apply to
improvements to real property which consists of six or fewer residential units which are under construction
simultaneously." 73 P.S. § 503(a).

8. Defendant owns only one property, which is not residential.

9. Defendant did not have six or more residential units under construction simultaneously.

10. Therefore, the Contractor and Subcontractor Payment Act is inapplicable to the instant matter,
rendering Plaintiff’s complaint legally insufficient.

WHEREFORE, Defendant respectfully requests that the complaint be dismissed for legal insufficiency
of a pleading for failure to state a claim for which relief can be granted pursuant to Pennsylvania Rule of Civil
Procedure 1028(a)(4).

Legal Argument

[Draft legal argument here.]

66
LIBRARY

67
592 A.2d 716 (Pa.Super. 1991)
Howard W.F. WOLFE, Jr., Appellant,

v.

Joseph B. PORTER, Appellee.

***

The dispute in this action of ejectment is between neighbors and pertains to the ownership of a seventeen (17)
foot wide strip of land along a common property line. Joseph B. Porter, the defendant, holds record title to the
disputed land. Howard W.F. Wolfe, Jr., the plaintiff, claims title by adverse possession. In order to establish his
claim, however, it is necessary that Wolfe tack his possession to a period in which his parents allegedly possessed
the land in dispute. The trial court held that tacking, under the facts alleged in plaintiff's complaint, was not
available to establish title by adverse possession and sustained defendant's preliminary objections in the nature of
a demurrer to plaintiff's amended complaint. After careful review, we affirm. [footnote omitted]

The standard of [trial and] appellate review was stated in Ward v. Serfas, [citation omitted] as follows:

When preliminary objections in the nature of a demurrer are filed, we must accept as true all the
well-pleaded material facts set forth in the complaint and all reasonable inferences deducible from
those facts. Accepting these facts and inferences, we then determine whether the pleader has failed
to state a claim for which relief may be granted, and we will affirm the grant of a demurrer only if
there is certainty that no recovery is possible. All doubts are resolved in favor of the pleader.
Furthermore, by filing preliminary objections in the nature of a demurrer, appellees have admitted
the factual allegations of the complaint for purposes of the demurrer.

[citations omitted].

***

A review of the facts alleged in the complaint . . . makes it apparent that plaintiff cannot establish title to the
disputed land by adverse possession.

"[O]ne who claims title by adverse possession must prove that he had actual, continuous, exclusive, visible,
notorious, distinct, and hostile possession of the land for twenty-one years." [citation omitted]. In the instant
case, the plaintiff-appellant has been in possession of the disputed tract of land since 1979. This is an insufficient
period to establish title by adverse possession.

Appellant argues, however, that when the period of his possession is tacked on to the period during which his
predecessor in title held possession, a continuous and adverse possession for more than 21 years is established.

The applicable law was stated in Wittig v. Carlacci, [citation omitted] as follows:

***

The possession of successive occupants may be tacked, but only where there is privity between
them. For our purposes, "privity" refers to a succession of relationship to the same thing,

68
whether created by deed or other acts or by operation of law.

But a deed does not of itself create privity between the grantor and the grantee as to land not
described in the deed but occupied by the grantor in connection therewith, although the grantee
enters into possession of the land not described and uses it in connection with that conveyed * *
* The deed, in itself, creates no privity as to land outside its calls. Nor is privity created by the
bare taking of possession of land previously occupied by the grantor.

Our court has held that acceptance of a deed describing boundary lines confined the premises to
the area within the boundaries, and that such a deed did not convey inchoate [meaning: “an
interest in real estate which is not a present interest, but which may ripen into a vested estate, if
not barred, extinguished, or divested.” Black’s Law Dictionary, Sixth Ed. (1990)] rights
acquired by incompleted adverse possession. [citation omitted] Each predecessor must have
claimed title to the property in dispute, and in transferring to his successors must have purported
to include it.

[citations and footnote omitted]

Thus, a grantee cannot tack his grantor's possession of land when the grantor does not convey such
land to him. The only recognized exception occurs where an intent to convey more land than that
described may be inferred from the circumstances or the deed itself.

[citations omitted]

Plaintiff-appellant's title to his land was acquired by deed from his mother, Catherine M. Conrad, formerly
Catherine M. Wolfe, on July 31, 1979. Prior thereto, the land had been owned by plaintiff's parents, Howard
W.F. Wolfe and Catherine M. Wolfe, husband and wife, who had acquired title by deed in 1952. Howard W.F.
Wolfe died April 27, 1978; whereupon, Catherine M. Wolfe became the sole owner thereof. Plaintiff's parents,
it is alleged, were in continuous possession of their own land and also of the disputed tract between 1952 and
1979. There is no suggestion, however, that title by adverse possession was perfected by adjudication or, indeed,
that a claim of title by adverse possession was ever made against the owners of the adjoining tract.

When Catherine M. Wolfe (Conrad) conveyed to her son, she conveyed only the land for which she had record
title. The deed contained a metes and bounds description of the land; and this description did not include any part
of the land now in dispute. Where title by adverse possession is inchoate, a deed by a grantor which fails to convey
such inchoate right is ineffective to create privity which allows tacking. The grantor's failure to include the
disputed tract in the 1979 conveyance to her son prevented the son from tacking his possession of the disputed
tract on to that of his parents to establish his own title by adverse possession. [citation omitted]

Appellant argues that an intent to include the disputed tract can be inferred from the language of the deed. We
disagree. A careful examination of the deed discloses no reference to the disputed tract or to any inchoate right
thereto. "The deed, in itself, creates no privity as to land outside its calls." [citation omitted] In the absence of a
conveyance of the grantor's inchoate rights, we assume that appellant's predecessors either occupied the disputed
land permissively or that they abandoned every intention of holding adversely before the conveyance. [citations
omitted] (where grantor had claim of adverse possession to certain tract of land, and subsequent deed to grantee
specifically excluded that tract, grantee was precluded from asserting continuing claim of adverse possession,

69
since he cannot be said to be in privity with grantor); [citation omitted] (although party claiming title by adverse
possession may tack possession of predecessors, party must show sufficient privity between them and that each
of them has transferred to his successor in some lawful manner the adverse possession or inchoate interest held).

***

As we have observed, appellant's own possession falls short of the required twenty-one (21) year period and the
amended complaint discloses that there is no privity between appellant and his predecessors in title which would
enable him to tack his period of possession to the possession of his predecessors. Therefore, appellant has failed
to state a claim for which relief may be granted, and the trial court properly granted appellee's preliminary
objection in the nature of a demurrer.

Affirmed.

70
812 A.2d 558 (Pa. 2002)
Frank A. ZEGLIN, Jr., and Tammy Lee Zeglin, Appellants,

v.

Sean E. GAHAGEN and Kimberlee H. Gahagen, Appellees.

***

In this appeal involving a boundary dispute, the question presented is whether privity of estate between succeeding
landowners is required to support tacking periods of ownership to form the requisite twenty-one-year period under
acquiescence theory.

Appellants, Frank and Tammy Zeglin, and Appellees, Sean and Kimberlee Gahagen, own adjoining properties in
Windber, Paint Township, Somerset County. The Zeglins purchased in 1977 from Cora Murphy, who, together
with her late husband, had owned the property since 1937. The Gahagens bought from Margaret Swincinski in
1989, who had acquired the parcel in 1979 from the previous owners since 1972.

In 1995, the Gahagens employed a professional to survey their property and learned that their deed described a
boundary on the Zeglins' side of a line marked by a row of bushes, utility pole, and fence that had been added by
the Zeglins. The surveyor therefore concluded that the Gahagens' property extended over such visible line, and
this was confirmed in a subsequent survey commissioned by the Zeglins. The Gahagens notified the Zeglins that
a portion of their driveway encroached on their land, removed the bushes, and constructed a retaining wall
adjacent to the surveyed boundary. The Zeglins responded by filing a complaint against the Gahagens sounding
in ejectment and trespass and claiming ownership up to the line previously demarcated by the bushes, utility pole,
and fence. In furtherance of this position, the Zeglins relied, inter alia, on the doctrine of acquiescence in a
boundary, alleging that their occupancy and possession, together with that of their predecessors in title, for a
period of more than twenty-one years established the visible line as the legal boundary. The Gahagens filed an
answer and counterclaim.

In March of 2000, following a non-jury trial, the common pleas court issued a decree nisi in favor of the Zeglins,
which it later made final. In accompanying opinions, the court summarized the acquiescence doctrine as follows:

an occupation up to a fence on each side by a party or two parties for more than 21 years, each
party claiming the land on his side as his own, gives to each an incontestable right up to the fence,
whether the fence is precisely on the right line or not; and this is so although the parties may not
have consented specifically to the fence in question.

[citations omitted]. The court identified as the basis for the principle public policy favoring peace and the repose
of titles. It reasoned that, for a period of more than twenty-one years, the Zeglins, the Gahagens, and their
predecessors in interest had recognized and acquiesced in a boundary line demarcated by the hedgerow (and also
highlighted by the fence maintained by the Zeglins through a portion of that time period). Although the Zeglins
had occupied the property for only eighteen years prior to the Gahagens' actions, the court permitted them to tack
the period of ownership by the Murphys, despite the fact that Cora Murphy had not specifically and formally

71
conveyed her purported interest in the disputed tract to the Zeglins in the written deed. As pertains to tacking
under the doctrine of adverse possession, the court recognized the requirement in Pennsylvania of privity of estate,
namely, a higher degree of relation than that of mere grantor and grantee of a main parcel, generally comprised
of specific and formal conveyance of the predecessor's interest in the disputed tract where the transfer is between
unrelated parties. [citations omitted] The common pleas court found, however, that Pennsylvania courts had
distinguished acquiescence in a boundary by applying the less rigorous requirement of privity of possession to
claims predicated on such theory. [citation omitted] ("Pennsylvania courts have adopted the view that succeeding
owners of property are bound by the fences that were accepted and recognized by former owners even without
any other privity or formal transfer of the area possessed adversely." [citation omitted]).

On the Gahagens' appeal, the Superior Court reversed . . . . The court determined, however, that, just as in the
case of adverse possession, privity of estate is an essential prerequisite to employment of tacking to perfect a
claim under acquiescence theory. [citations omitted] Accordingly, the Superior Court held that the common
pleas court erred by permitting the Zeglins to tack the period of the Murphys' ownership based on privity of
possession alone. [citation omitted]

Presently, the Zeglins argue that privity of estate as a prerequisite to tacking is inappropriate to, and contrary to
the doctrine of, acquiescence in a boundary, since an underlying premise of such theory is that the evidence of
longstanding acquiescence in a physical boundary by adjoining property owners will control over contrary deed
calls. The Zeglins distinguish Plott v. Cole, cited by the Superior Court, as allowing for creation of privity by
"other acts," and not solely by references culled from a deed. The Gahagens concede that the privity of estate
requirement has not expressly been attached by Pennsylvania courts in acquiescence cases, but contend that such
a requirement would alleviate confusion among landowners.

The establishment of a boundary line by acquiescence for the statutory period of twenty-one years has long been
recognized in Pennsylvania. [1] Two elements are prerequisites: 1) each party must have claimed and occupied
the land on his side of the line as his own; and 2) such occupation must have continued for the statutory period of
twenty-one years. [citation omitted] As recognized by the Superior Court and the common pleas court, the
doctrine functions as a rule of repose to quiet title and discourage vexatious litigation. See id. at 592, 547 A.2d
at 1220.

***

As President Judge Coffroth [of the Somerset County Court of Common Pleas] aptly observed, the reason why
privity of estate should not be deemed necessary to support tacking in this setting is, simply, because a prospective
purchaser will see the fence or similar marking; given its "obvious presence as apparent boundary," he is therefore
put on notice to inquire about its origin, history, and function. [citation omitted] [5] [12] ("After 21 years, the
chips will be allowed to fall where they may, for reasons of equity and peace.").

Accordingly, we find the majority view (requiring only privity of possession) better suited to claims brought
under a theory of acquiescence in a boundary. We hold, therefore, that tacking is permitted in such context upon
sufficient and credible proof of delivery of possession of land not within (but contiguous to) property described

72
by deed of conveyance, which was previously claimed and occupied by the grantor and is taken by the grantee as
successor in such interest.

The order of the Superior Court is reversed, and the case is remanded. . . .

***
Notes:

[1] See Reiter v. McJunkin, 173 Pa. 82, 84, 33 A. 1012 (1896) ("After 21 years of occupancy up to a fence on
each side as a line fence, it is not material to inquire whether the fence is on the right line or not."); [citations
omitted] ("a boundary line may be proved by a long-standing fence without proof of a dispute and its settlement
or compromise") [citation omitted].
***
[5] "[A]cquiescence" in the context of disputed boundaries "denotes passive conduct on the part of the lawful
owner consisting of failure on his part to assert his paramount rights or interests against the hostile claims of the
adverse user." [citation omitted] . . . .
***
[12] [citations omitted] (indicating that "color of title [is] not necessary for tacking to provide continuity of
possession of land, provided the land in question [is] contiguous to that described in a deed, and that lands both
titled and untitled were part of a close, apparent by reason of physical boundaries such as fences or hedges."
[citations omitted]

73
Instructions

The performance test is designed to test an applicant’s ability to perform the legal
task that has been assigned using the factual information contained in the File and
legal principles that are provided in the Library.

The File contains the only factual information that you should consider in
performing the assigned task. The task to be completed is set forth in the first
document in the File in the form of a memorandum to the applicant. The Library
contains the only legal principles that you should consider to complete the assigned
task. Although your general knowledge of the law may provide some background
for analyzing the problem, the factual information contained in the File and the
legal principles contained in the Library are the only materials that you should use
in formulating your answer to the assigned task.

Your response should be written in the gray answer book or typed in answer screen
number 3 of SofTest. Be sure to allow sufficient time for reading the materials,
organizing your answer and completing the task assigned. Your answer should
demonstrate an understanding of the relevant facts, recognition of the issues and the
applicable principles of law and the reasoning that supports your answer. Your
grade will be based on the content of your response and your ability to follow
instructions in performing the assigned task.

The events depicted and the persons portrayed by the information in the File are
fictitious and such information does not depict nor is it intended to depict or portray
any actual person, company or occurrence. Any similarity to any person, living or
dead, or any occurrence is purely coincidental.

74
Question No. PT: Examiner’s Analysis

The applicant is assigned to draft preliminary objections to a complaint that seeks to quiet title via adverse
possession and consentable/acquiescence in a boundary; and to draft the legal argument section of a brief in
support of those preliminary objections. Applicants are instructed to limit their preliminary objections to a
single demurrer to each of the two counts of the complaint. The caselaw provided should lead applicants to
focus their demurrers on: 1. The inability of the plaintiff to “tack” his claim onto his predecessor’s claim to
meet the 21 year time limit under the theory of adverse possession; and 2. The concept of
consentable/acquiescence in a boundary is inappropriate because our case does not involve a boundary dispute,
as there is no identifiable boundary line in question.

Drafting the Preliminary Objections 3 Points

The applicant should follow the directions in the assignment memorandum and formatting memorandum and, in
formatting the preliminary objections, should include separately numbering paragraphs, address each objection
in a separate section, and draft both preliminary objections and a legal argument section of a brief.

The applicant should identify the following two preliminary objections, and set forth the basis for those
objections:
1. Plaintiff’s complaint is legally insufficient because it fails to state a valid cause of action for
adverse possession where the Plaintiff has not possessed the tract in question for the required 21
years and cannot “tack” on his predecessor’s purported use;
2. Plaintiff’s complaint is legally insufficient because it fails to set forth all elements of
acquiescence in a boundary, specifically that there exists no actual boundary.

The applicant should also address the two preliminary objections in a legal argument section that the managing
partner will include in his brief in support of preliminary objections. The analysis for the preliminary objections
and legal argument should be as follows:

Plaintiff’s complaint is legally insufficient because it fails to state a valid cause of action for adverse
possession because he has not possessed the tract in question for 21 years and he cannot “tack” on
purported use of the tract in question by his predecessor in interest. 9 Points

Pennsylvania Rule of Civil Procedure 1028(a)(4) permits preliminary objections to be filed to any pleading on
the basis of legal insufficiency (demurrer). Formatting Memorandum.

The trial court’s standard of review of a preliminary objection in the nature of a demurrer is as follows:

When preliminary objections in the nature of a demurrer are filed, we must accept as true all the
well-pleaded material facts set forth in the complaint and all reasonable inferences deducible
from those facts. Accepting these facts and inferences, we then determine whether the pleader
has failed to state a claim for which relief may be granted, and we will affirm the grant of a
demurrer only if there is certainty that no recovery is possible. All doubts are resolved in favor
of the pleader. Furthermore, by filing preliminary objections in the nature of a demurrer,
appellees have admitted the factual allegations of the complaint for purposes of the demurrer.

75
Wolfe v. Porter.

“One who claims title by Adverse Possession must prove that he had actual, continuous, exclusive, visible,
notorious, distinct, and hostile possession of the land for 21 years.” Wolfe v. Porter.

Plaintiff cannot meet all elements of adverse possession for the requisite 21 year period. In order to do so,
Plaintiff would have to have possessed the tract in question for twenty one years before he could successfully
assert a claim for adverse possession.

Plaintiff acquired Lot 3 in the Lake Anne Subdivision on February 1, 2008. Complaint at Exhibit A.

Because Plaintiff has only owned Lot 3 for a little more than ten (10) years, he would have to rely on the legal
concept of “tacking” in order to arrive at the required 21 year time period for adverse possession to be
recognized. Complaint and Wolfe v. Porter.

In order for tacking to apply, “[e]ach predecessor must have claimed title to the property in dispute, and in
transferring to his successors must have purported to include it.” Wolfe v. Porter.

“Thus a grantee cannot tack his grantor’s possession of land when the grantor does not convey such land to
him.” Wolfe v. Porter.

When a deed of conveyance contains a metes and bounds description of the land conveyed and said description
does not include the land in dispute tacking cannot be established in an adverse possession claim. Wolfe v.
Porter.

The deed attached to plaintiff’s complaint at Exhibit A for Lot 3 contains a metes and bounds description for
Lot 3, and transfers title to only Lot 3 from an Ethan Bishop to Plaintiff, Paul Kerrigan. Complaint at Exhibit
A.

Nowhere in plaintiff’s deed is there mention of the portion of defendant’s property that is across the public dirt
road from Lot 3; nor does the conveyance make any reference to the tract of land over which Plaintiff claims
adverse possession.

To the contrary, the tract in question is a part of the property deeded to defendant. Complaint at Exhibit B.

In a claim for adverse possession “[t]he possession of successive occupants may be tacked, but only where there
is privity between them.” Wolfe v. Porter.

“Privity refers to a succession of relationship to the same thing, whether created by deed or other acts or by
operation of law . . . the deed, in itself, creates no privity as to land outside its calls. Nor is privity created by
the bare taking of possession of land previously occupied by the grantor.” Wolfe v. Porter.

A deed does not of itself create privity between the grantor and the grantee as to land not described in
the deed but occupied by the grantor in connection therewith, although the grantee enters into possession
of the land not described and uses it in connection with that conveyed * * * The deed, in itself, creates
no privity as to land outside its calls. Nor is privity created by the bare taking of possession of land
previously occupied by the grantor. Wolfe v. Porter.

76
Rather acceptance of a deed describing boundary lines confines the premises conveyed to the area within the
boundaries, and such a deed does not convey inchoate rights acquired by incompleted adverse possession. Wolfe
v. Porter.

The absence of any mention, or even a hint of reference, to Lot 3 in Plaintiff’s Deed demonstrates that the
predecessor in title to Lot 3 did not purport to include the tract in question in the transfer to Plaintiff. Therefore,
there is no privity between the successors in title to the parcel across from Lot 3.

Plaintiff is not in privity with prior owners in the chain of title because the deed conveying his property rights
from 2008 to present made no reference to the tract across the road from Lot 3. Therefore, he cannot “tack” on
any purported use or claim of possession by prior owners. Thus, Plaintiff has failed to demonstrate the
necessary elements of adverse possession existed for at least 21 years, and his Complaint must be dismissed.

Plaintiff’s complaint is legally insufficient because it fails to state a valid cause of action in that it has
failed to demonstrate all elements of acquiescence in a boundary. 8 Points

Plaintiff’s complaint attempts to acquire ownership of the 40’ x 50’ tract directly across the public dirt road
from Lot 3, and which is a part of Lot 6, by “consentable/acquiescence in a boundary.” Complaint.
Acquiescence is a legal concept that addresses boundary line disputes between adjoining landowners. See
generally, Zeglin v. Gahagen.

“The establishment of a boundary line by acquiescence for the statutory period of twenty-one years has long
been recognized in Pennsylvania. Two elements are prerequisites: 1) each party must have claimed and
occupied the land on his side of the line as his own; and 2) such occupation must have continued for the
statutory period of twenty-one years.” Zeglin v. Gahagen.

Plaintiff has failed to identify the essential factor in an acquiescence claim: a disputed boundary line.

Plaintiff has failed to identify any mark, such as a tree line, fence, hedges, etc., that constitutes a boundary line
of the tract across the public dirt road from Lot 3. To the contrary, plaintiff’s complaint indicates that
defendant’s lot (Lot 6, containing the tract in question) is a large open field, with no fencing, trees or other
distinguishing characteristics. Complaint, paragraph 6.

“[C]olor of title [is] not necessary for tacking to provide continuity of possession of land, provided the land in
question [is] contiguous to that described in a deed, and that lands both titled and untitled were part of a close,
apparent by reason of physical boundaries such as fences or hedges.” Zeglin v. Gahagen.

A boundary line may be proved by a long-standing fence without proof of a dispute and its settlement or
compromise. Zeglin v. Gahagen.

Because a prospective purchaser will see a fence or similar marking, given its obvious presence as apparent
boundary, he is . . . put notice to inquire about its origin, history, and function.” Zeglin v. Gahagen.

There simply is no boundary line presented by plaintiff upon which a consentable/acquiescence in a boundary
claim could be brought. The only boundary line identified in the complaint is the public dirt road between Lot 3
and Lot 6, and that is not the disputed line. Rather, plaintiff claims land on the defendant’s side of the public
dirt road, not on his own side of that boundary line, making a claim for acquiescence in a boundary improper.

77
Further, “‘acquiescence’ in the context of disputed boundaries ‘denotes passive conduct on the part of the
lawful owner consisting of failure on his part to assert his paramount rights or interests against the hostile
claims of the adverse user.’” Zeglin v. Gahagen.

Plaintiff’s complaint gives no indication that defendant has failed to assert his paramount rights or interests
against plaintiff’s claim, other than an implication that defendant has not objected to plaintiff’s use.

Because plaintiff has not established that he or his predecessors in title established any form of boundary lines
to distinguish the tract in question throughout the past 21 years, he cannot claim that the defendant acquiesced
to his possession of the tract. Thus, he has failed to demonstrate the necessary elements of acquiescence in a
boundary, and his complaint must be dismissed.

Furthermore, Pennsylvania caselaw dealing with the concept of acquiescence in a boundary generally addresses
situations in which a purported boundary has been established and agreed upon by the parties, and one party
claims ownership of the property up to the boundary line. Zeglin v. Gahagen. Here, as set forth above, the only
conceivable boundary line in question is the public dirt road. However, plaintiff is not simply claiming
ownership of the land up to the road; he is claiming ownership of the land for fifty (50) feet beyond the
boundary line. Thus, acquiescence in a boundary cannot apply in this instance, and plaintiff’s claim should be
dismissed.

78

You might also like